Compilation 509+ MCAT Study Habits

This forum made possible through the generous support of SDN members, donors, and sponsors. Thank you.

supremus

Full Member
7+ Year Member
Joined
Jan 13, 2015
Messages
86
Reaction score
77
Use the template below:

1) Your individual scores and composite score
2) The study method used for each section
3) What materials you used for each section(Kaplan, TPR, Examkrackers, AAMC, TBR, etc)
4) Which practice tests did you use? (Optional: include scores)
5) What was your undergraduate major?
6) Any other tips you may have for those of us who still have this test lurking over us?
7) How long did you study for the MCAT?

Members don't see this ad.
 
Last edited:
  • Like
Reactions: 28 users
Solution
Use the template below:

1) Scores:
  • C/P: 128
  • CARS: 131
  • Bio: 132
  • Psych: 127
  • Total: 518
2) The study method used for each section
  • Overall: I read every page of the review books and took notes for chem and psych, as they were my weakest sections. For any section that I had never gone over in school, I read and took notes (such as the metabolism sections). I would go back and forth with the Kaplan online videos/lessons, their practice problems, and the book in order to really memorize the info
  • Chem/Phys: this was easily my weakest section to begin with, so I went back to the basics (for chem)
  • CARS: the Kaplan tips on how to work with the CARS section were invaluable. Their hour-long MCAT channel...
1) Your individual scores and composite score:
Total: 523 PS: 130 CARS: 131 BS: 132 Psych: 130

2) The study method used for each section
PS: I used TPR for heavy content review and took detailed notes on each section. I also used their practice sections apart from practicing on the full length tests. Making an equation sheet as I went was really helpful so that I could look over it every other night just to ultimately memorize them.
CARS: The only way to really improve on this section is to do a whole bunch of practice passages. I didn't like TPR method and instead used it as a starting point. Though it sounds counterintuitive, I decided to go about it by not reading the whole passage and rather skimming. My score took a hit when I first started but quickly improved once I had it down. I was consistently getting 126 on CARS sections and then improved to regularly getting 130 once I changed my strategy. I used the Exam Krackers practice sections for this along with the AAMC question backs (though I didn't find them as helpful oddly).
BS: Honestly, researching helped a lot with this. And if you don't research, any lab course where you can do some bench work would really help on this section. I used TPR for review with this and also just the notes I had from my pre-req classes.
Psych: A lot of people said that psych was going to be the easiest section and I guess it was easier but I had a hard time improving just because there were always words I didn't know. So, I borrowed all different company test prep psych books from my friends and would spend a day or two just looking through the glossaries. It really helped to learn words across different companies to get as much exposure to the section as possible.

3) What materials you used for each section(Kaplan, TPR, Examkrackers, AAMC, TBR, etc):
PS: TPR and class notes
CARS: EK, AAMC material, and a little TPR though TPR was a bit lengthy and not as representative
BS: TPR and NextStep
Psych: TPR, NS, Kaplan, EK, and AAMC stuff

4) Which practice tests did you use? (Optional: include scores):
AAMC FL1, 2, and 3: 506, 517, 523
TPR1 and TPR2: Scored 506 on all of these even though I took them before studying and after. They really depress your score
NS 1, 2, 3, 4, 5, 6, 7, 8, 9: 512, 514, 511, 511, 511, 509, 510, 512, 511
AAMC Question Packs:
Official MCAT Biology Question Pack 1
Official MCAT Chemistry Question Pack
Official MCAT Physics Question Pack
Official MCAT Biology Question Pack 2
Official MCAT Critical Analysis and Reasoning Question Pack Volume 1
MCAT AAMC Official Guide Questions

5) What was your undergraduate major?
Chemical and Biological Engineering with a minor in Biomedical Engineering

6) Any other tips you may have for those of us who still have this test lurking over us?

I suggest not breaking your time into studying content review and then practice tests. Interweaving the two from the get-go as you study really acquaints you with the testing format. That doesn't mean you need to take a full length every weekend, but I would study a few chapters from the books and then do like a bio section. This way, by the time I finished content review, I also understood the format of the test and could jump straight into the full lengths to gauge score. Also, I only took three practice tests where I sat down and took it in one go. I liked taking one practice test a week interspersed. It helped with not completely being exhausted and hating the MCAT by the end.

Timingwise, I timed everything shorter than the actual MCAT. Though it sucked always running out of time in the beginning, the actual MCAT was a lot more comfortable.

I took detailed notes when I read through TPR books and then never referred to the books again. Three weeks before the test, I would go through all my notes on every section once every few days and then every day as I approached the final week.

I agree with what someone else said that practice is more important than review, Unfortunately there will always be stuff you don't know on the MCAT and I've found that knowing how to get through really helps with attaining a high score. I took many many many practice tests and still saw a bunch of stuff on the MCAT I hadn't seen before.


7) How long did you study for the MCAT?
4 months. I took TPR bootcamp during my winter break and studied from Jan to April on top of 14 credits worth of hard engineering courses. I also volunteered, researched, and was part of clubs on top of a serious family medical emergency. It's pretty safe to say I had no life.

Side Note: There are a lot of downers in this world and a lot of people that will say you can't do it. The number of times that people told me I couldn't handle what I was doing was insane. Just gotta block the haters out and do your own thing. I think anyone can do well on the MCAT and am totally open to answering questions about study schedule or anything else if needed!
 
Last edited:
  • Like
Reactions: 6 users
I didn't like TPR method and instead used it as a starting point. Though it sounds counterintuitive, I decided to go about it by not reading the whole passage and rather skimming. My score took a hit when I first started but quickly improved once I had it down. I was consistently getting 126 on CARS sections and then improved to regularly getting 130 once I changed my strategy.

Could please elaborate on how you skimmed and what change you made in your strategy?

Thank you
 
Members don't see this ad :)
1) Your individual scores and composite score
Chem/Phys - 128, CARS - 128, Bio/Biochem - 128, Psych/Soc - 129

2) The study method used for each section
For every section except CARS, I read through the Kaplan books at least once, highlighting the important stuff. Then I printed out AAMC's MCAT outline and I read the books a second time, this time following the order provided on the outline. I found this second read to be more helpful because the AAMC's outline did a good job of listing related topics after one another.

I spent the most amount of time with the Biochem because because I hadn't taken biochem before I took the MCAT. Since the science sections are mostly biochem, it would probably be very helpful to take the class. But the Kaplan book did a very good job of telling you what you needed to know.

I didn't "study" for CARS at all, I figured I would just practice with the Kaplan tests.

After this, I went back and if there were some topics that I still didn't feel comfortable with, or just topics that I thought would be high yield on the test, I read them carefully. This was probably most helpful because there wasn't as much information this time around so I was able to spend a lot of time looking at these chapters.

Throughout this studying, I probably took a practice test once every 2 weeks, and towards the end I took one a week. The tail end of my prep was just taking practice tests/practice sections and reviewing my tests, including questions I got right, each night.

3) What materials you used for each section

I used the Kaplan 7 book review for all of the sections. I watched a few Khan Academy videos on metabolism when I didn't feel like studying, but this did nothing for me because I literally had the video open in one window and games open in the other. You could use it as passive studying if you're actually gonna pay attention I guess, but I didn't.

4) Which practice tests did you use?

I took about 6-7 Kaplan practice tests, the Kaplan diagnostic, a few Kaplan section tests, the free Next Step Prep half length diagnostic, the AAMC full length practice test, and a few random tests/questions I found online. I also did an MCAT question of the day each morning from MCAT Prep or something like that, but I found Kaplan's question of the day, which I didn't use until like the week before the test, to be way more relevant.

Although the scores I got from the Kaplan practice tests were not at all reflective of my real scores (practice scores were significantly lower for every section), I feel that it is currently the best material out there for the new test, considering the lack of material the AAMC has put out so far. Kaplan's questions are not straightforward at all and they try to trick you, but I found the style and length of passages from the Kaplan full lengths to be similar to the real test. It's a good way to familiarize yourself with the test format, but don't look too much into the scores.

5) What was your undergraduate major?
Bio

6) Any other tips you may have for those of us who still have this test lurking over us?

I have two major tips. The first is to spend more time practicing and less time with content review. There's no replacement for learning the content well, but with any standardized test, getting familiarized with the style and the question types you will receive goes much further than studying.

At the beginning I did mostly content review and didn't take many practice tests, and my scores either stayed stagnant or went down. Near the end I started taking more tests and kept up with content review, and I noticed that my scores steadily went up. Becoming familiar with the style and format will help more than simply trying to memorize the books.

It might be annoying to take a 7 hour test once a week or even multiple times a week, which is what I did at the end, and at these times I found section tests to be helpful. This still gave me good practiced and improved my test taking skills but it only took an hour and a half of my day. I would use individual section tests to focus on your weak sections. I did this mostly for CARS but also Chem/Phys.

The second major tip I have is RELAX. I think this is the biggest thing and it sounds cliche but it actually makes such a difference. Every time I finished a Kaplan test I freaked out because I was getting around 500 every time, sometimes below 500. As I reached the end of my prep, I did the AAMC practice materials and got more comfortable because these were much more straightforward than the Kaplan materials I was using.

I think a big thing for me on test day was that I was relaxed the whole time. I was kinda anxious when I got to the test center, but I used the 10 minute tutorial to calm myself down before beginning the test. I'm pretty sure this was the reason I performed relatively well. I never scored above 127 on any section of a practice test, and usually my scores were 124-125. On the real thing, I got 128 on 3 sections and 129 on the last. I think being calm and collected throughout the test helped me a lot.

7) About 2.5 months, about 5-6 hours a day of actual studying, and in the last month I added about an hour a day of reviewing old tests.

What company did you buy the individual section tests from?
 
Woo thought I'd do this while procrastinating on secondaries ehehe.

1) Your individual scores and composite score
Total: 523
PS: 130, CARS: 129, BS: 132, Psych: 132

2) The study method used for each section
AAMC banks for PS, BS, and Psych. Memorization is huge for the psych section unfortunately, but you also do need to understand how the terms apply etc.
TPR for CARS.

3) What materials you used for each section(Kaplan, TPR, Examkrackers, AAMC, TBR, etc)
I used the Kaplan set of subject books for content review and I bought a TPR book too but rarely used it. I did use the online TPR practice CARS sections. Other than that, only AAMC tests and section banks. Oh, and 1 free NS FL.

4) Which practice tests did you use? (Optional: include scores)
NS 1 (the free one): 512
AAMC 1: 515
AAMC 2: 516
AAMC unscored: Unsure
AAMC 3: 523

5) What was your undergraduate major?
Essentially molecular biology.

6) Any other tips you may have for those of us who still have this test lurking over us?
Know yourself and your strengths/weaknesses! I mean, I think this thread in itself may go against that, but really reflect on your learning and what you have struggled with etc. It’s much easier to up your “weak” sections than to strive for a perfect score on your strong sections, as in the higher score ranges you’re talking a 1-2 question difference for each of the points. Don't go just off of what other people say is the "best strategy," find your best strategy... :p

Personally, this might be a bit unconventional, but what I did was to divide the studying up into 2 “phases.” The first part I did all of the “deep” learning (things you really need to understand and that I knew I struggled with) such as physics and ochem. Also CARS is a good one, as it takes a lot long term experience with reading comprehension. During this time I ignored the “memorization” stuff, just focusing on understanding the concepts rather than memorizing.

Then 2-3 weeks before I switched gears and did the memorization heavy stuff. Formulas, units, intermediates in the biochem metabolism pathways, amino acids, psych terms. This resulted in the score jump from 516->523, although I’m not sure what I scored in the middle there.

And then finally don’t neglect to take lots of FL practice tests and spend as much time taking them as you do going over them. Every single question that you were a little unsure on, go over it and figure out why you were unsure. Don’t wait to take FLs, take one at the beginning of your studying so you know where you stand, and gauge your studying progress by taking one every so often.

7) How long did you study for the MCAT?
3 months
Although I feel like this question is deceiving because I took it in spring of my junior year. I’d just finished most of the prereqs within the last 1-2 years and knew that most of it was pretty fresh. If that’s not you, take a longer time to really understand the material and ensure you're confident with the test. No shame.

Honestly did not expect this test to go as well as it did but it was a hell of a journey getting there (just crossing my fingers for this admissions cycle now lol). Feel free to PM with any questions.
Hey L congratulations and thanks for taking the time to post this. How did you study for bio though?
 
I did not study much of the molecular biology stuff as that is my major and I am quite familiar. In general, techniques will come up a lot, so know PCR/gel electrophoresis/western blot/SDS-PAGE/types of column chromatography/immunofluorescence assays, not just what they do but how they work. For biochemistry, it is mainly memorization but also realizing the "big pictures" of each metabolic pathway and how they are regulated. Amino acids are a must know (3 letter and 1 letter codes, structure & important features, whether they are hydrophobic or hydrophilic). There's a fun amino acid matching game online that I'd play every now and again to refresh my knowledge. For physiology, I actually felt like I neglected a lot of the "memorization" stuff but what I did focus on was a couple "hot topics" and really understanding how they work/are regulated. Just off the top of my head: the renal filtration system and its regulation by ADH and aldosterone, the menstrual cycle and its regulation by progesterone/estrogen/hCG/other hormones, neural action potential, the muscle sarcomere contraction cycle, blood carbonate buffer system, basic immunology.
 
  • Like
Reactions: 1 users
Could please elaborate on how you skimmed and what change you made in your strategy?

Thank you
When I read the whole passage, I was able to accurately answer questions but never finish all of them so my score kind of hit a plateau. My new strategy helped me overcome that. I fully read the first and last paragraph, but would only read the first and last sentence (ish) of each paragraph in between. That helped me determine the main idea and then I'd answer the questions based on which choice best fit the main idea. Retrieval questions were the toughest with this method, but I felt like even those answers had more to do with the main idea than the nit-picky details. It's a weird way to go about CARS but somehow worked incredibly well!
 
  • Like
Reactions: 1 user
1) Your individual scores and composite score:

Total: 519 C/P: 131 CARS: 129 B/B: 131 P/S: 128

2) The study method used for each section

So I want to preface this by saying that I just completed a postbacc where I took all the prereqs and then took the MCAT actually in between ochem 1 and 2 (took them both as summer sessions--never again). So my content review and content learning happened at the same time for a lot of the material.

C/P: I focused mostly on concepts and practice passages/discretes. I watched a lot of Khan Academy videos on the chemistry and physics topics, and I read a lot of hyperphysics and other things (partially for pleasure, but partially to study) as well. I think I made an equation sheet once, but my gen chem professor was really big into not memorizing equations if it wasn't necessary and just focusing on unit analysis, so that's what I ended up doing. I spent more time making sure I knew all the units for different things (e.g., that a Newton is kg*m/s^2 or that a Pascal is a kg/m*s^2). That way, even if I didn't know a formula or whatever, just by knowing what I was trying to get and the units of what I had, I could usually get to the right answer. I used UWorld for about a month before the exam using the free trial.

CARS: I kind of shot myself in the foot here. A 129 is a good score, but I was getting 131s and 130s on my AAMC FLs, and my verbal section on the GRE and SAT were nearly perfect. Because of all that, I did not really do any CARS preparation. I did the first Q bank like 6 months before the test, then stopped because my FL scores were so good. The real CARS was way harder than any of the FLs, and while I can't be sure it would have helped, I think if I had done more passages, my score might have been higher.

B/B: I did not take biochemistry prior to taking the MCAT, so I had to teach myself a lot of biochem. I listened to Kevin Ahern's podcast and some videos on youtube, but what I realized very quickly was that most of the biochem passages had clues to the right answers in the passage themselves, with very few pseudodiscretes. Therefore, I focused far more on figuring out the patterns within the passages and using the critical reasoning skills I learned getting my math degree to parse out the answers rather than trying to teach myself an entire semester of biochem while also taking 16 credits of science courses followed by a summer session of ochem. I did still try to learn the content, but I focused on high yield stuff that could get me multiple answers without having to memorize a ton of stuff.

P/S: I hate psych/soc and did not study at all. I read about 30 pages of the Khan Academy 100-page document and decided that I absolutely hated how they would take a word with a common definition and then define it in a completely different way seemingly for no reason. So yeah. I did not study at all and found myself able to guess my way to a 128 a lot of the time. 0/10 would not recommend for most people.

3) What materials you used for each section(Kaplan, TPR, Examkrackers, AAMC, TBR, etc):

I did an in-person Kaplan course paid for by my postbacc. It was somewhat useful.

C/P: Kaplan, KA, hyperphysics, UWorld, class notes
CARS: AAMC CARS Qbank 1
B/B: Kaplan, KA, MedSimplified (a bomb ass YouTube channel), UWorld
P/S: Virtually nothing

4) Which practice tests did you use? (Optional: include scores):

AAMC FL1, 2, and 3: 516, 513, 515
Kaplan 1, 2, 7: 503, 507, 507
NS 1, 2, 3: 512, 511, 511
AAMC SBs and all Qbanks (except CARS vol 2)


5) What was your undergraduate major?

Mathematics with a minor in English

6) Any other tips you may have for those of us who still have this test lurking over us?

Start taking practice tests early. I would not do more than one every two or three weeks to start with, but getting familiar with the test and the stamina required is important. It's like training for a marathon. You need to build up the stamina. You do not need to take a hundred practice tests. You will burn out that way. I suggest taking 3 (4 tops) FLs under true testing conditions--at the same time, with the same food, keeping the same rules in effect. That will familiarize you with what it's like and build your stamina up.

I unfortunately did not have to opportunity to take all the prereqs and then do content review, so I was doing content review for courses I had taken the previous semester while learning the material from the next semester from scratch simultaneously. It kind of made things a little more convoluted. However, I think it is smart to do content review and practice simultaneously. If you already have learned most of the material, doing practice tests and qbanks and stuff early on will give you a good indication of your weak areas that you really need to focus on.

I took all my FLs timed, but I did all of the SBs and Qbanks untimed. I think learning how to reason through the passages is really important. If it takes you longer at first, that's fine. You'll get faster as you practice.

Others have said it, but I'll say it again. You will not know everything that could be on the exam. It's just not possible. Focus on practice and on how to read the passages and reason through the gaps and clues. That will help you be more flexible and get more points.

7) How long did you study for the MCAT?

I did actual MCAT studying for about 5 months.
 
  • Like
Reactions: 1 users
How do you guys space out CARS practice ~3 weeks out from test date? I just bought the AAMC CARS 1 and 2, which have 120(?) questions each. But apparently you can only "start" it 5 times.
 
Two words: Youtube videos and Uworld.

1) 514 (128, 125 (!), 129, 132) 91 percentile. First time taking it.

FL 1: 512
FL 2: 515
FL 3: 511

Background: 3.4 GPA, 3.27 science. I went to a public research university (not a top 20). Lots of B's in science classes (and C in ochem+biochem). I felt like I understood the science content but for some reason wouldn't test well. If you struggled with science classes do not worry, if I did it you will be able to :)

Classes that (I think) helped me for the test: physiology (even tho I had to relearn a lot of it), biochem, cell biology.

2)
C/P: Oh boy this is my worst section. I read all of the EK books for this and it was very helpful, however, the material FINALLY clicked when I started watching Pre-Med HQ videos on youtube. His videos were straight to the point and so concise that it finally made sense. This guy needs more viewers and is so underrated. I tried to watch AK and KA but the videos were so long and painfully detailed that I would get lost. After content I practiced LIKE CRAZY using all the AAMC q packs and section banks. After I finished those I worked on the uworld passages and finished all of physics and ochem questions. Practice is key here and doing scientific notations for the calculations.

B/B: This section is usually my strength and I ranged from 130-131 on the AAMC tests. I read the EK books and did a lot of practice problems. I did the section bank 3 times and that really helped me on test day (I def think you should redo these at least 3x for each section and try to not use prior knowledge when answering it). I started to see patterns and on test day I could anticipate what questions they were going to ask from the passage. Also EK didn't cover metabolism in detail so I would recommend watching YT videos on glycolysis, Krebs, ETC and to less detail gluconeogenesis, glycogensis, etc (have a good idea but don't kill yourself with small details, just know the use and the important enzyme for each).

CARS: I suck at this because english is not my first language. CARS I practiced reading slowly and then answering questions (but obviously it didn't help on test day). I didnt really see improvements while I studied so I started to emphasize this section less. Test day was the lowest I scored (I usually got around 127-128 on practice AAMC). The weird thing is that on test day the passages were the easiest I had read. Oh well!

P/S: This was the lowest scored section while I was practicing. 5 days before my test I scored 127 and a week before 126! The last 5 days I reread the 86 page document and did ALL the uworld P/S questions over the last 3 weeks which SAVED my ass test day. If I didn't do uworld there's no way I would have gotten a 132.


4) I used NS 1-4 and ranged from 507-509 at the end of my studying (I took them a few months prior but didnt look at solution, I was scoring 500-503 then). I would use them again even though they were tricky. They made AAMC feel like a breeze once I switched over to the AAMC practice content.

5) Biology. I was a Biology teacher too which helped with my foundation (mitosis/meiosis, evolution, organelles, punnett squares, etc)

6) Practice for at least 6-8 weeks, you'll be reviewing content anways as you practice. I feel that content is 40% and is 60% practice. So content is pretty important but not if you can't apply it to the test. Through practice you get comfortable seeing which questions will be asked and some of the things you need to learn aren't explicitly taught in the books. Use reddit when reviewing your tests (the solutions in AAMC suck). Don't take the test until you're ready. I rescheduled my test 3x until I felt ready.

Also, test in testing conditions. I would get anxious and nervous during my practice tests so I practiced breathing and meditation before the tests. I'm not a yogi or anything but it helped on test day because I was super calm and focused. Also go to the testing center the day before to calm your nerves and get familiar with the site. And for the love of god SLEEP! I got a good 7 hours of sleep the night before. I can't imagine not sleeping like some of these other testers. Also during the test if you're spending too long on the questions flag it and move ON! You will have extra time at the end and you can go back and look with a fresh pair of eyes. I did this on test day and I was able to figure out the problems quickly once I took a little break.

7) Started in January so 5 months part time, few hours M-F, 5-6 hours on weekends (I was working 40-50 hours a week). I took a break in March for a week to travel. From May to end of June I studied 1 month full time.


Don't stress, take your time, YOU CAN DO IT.
 
Last edited:
  • Like
Reactions: 3 users
1) Your individual scores and composite score

129/129/129/128 (515) -- August 2017

2) The study method used for each section

No conscious study method. I just really like science. And analyzing problems. I took the MCAT as a challenge to prove my math and science teaching ability to other people as an organic chemistry tutor and I actually only became serious about pursuing medicine after I got my scores (I had a total uGPA of 3.17)

3) What materials you used for each section(Kaplan, TPR, Examkrackers, AAMC, TBR, etc)
Kaplan review books I bought on eBay, especially for CARS and P&S, and Khan Academy for CARS and P&S
4) Which practice tests did you use? (Optional: include scores)
Next-step MCAT, free half-length assessment and full-length assessment
5) What was your undergraduate major?
Biochemistry
6) Any other tips you may have for those of us who still have this test lurking over us?
Teach math and science to high school students and college students. Students will ask you garden path questions and you have to be two steps beyond them. You quickly start getting pressured to know your ****. When you teach, you learn even more. I feel like most of my content knowledge came from teaching. The books just taught me how to use that content knowledge effectively on the test (i.e. test-taking strategies).

Proper sleep before the test. Bring lunch and snacks. The test is long and you need the energy. I feel like I would have done better if I hadn't been sleep-deprived and spent the previous night in the ER where I had to watch an arrested man under police custody scream as the nurses put him in four point restraints.

Also, because mental math is emphasized at times and you find yourself taking the square root of numbers like 1.0 * 10^-9 or 7.0 * 10^-8, knowing the sqrt(5) or sqrt(7) by heart would have helped me on a couple questions because I ran out of time on the first and fourth sections. I should have focused less on content and more on time management.

7) How long did you study for the MCAT?

I literally spent 2 days reviewing CARS and testing myself on sample questions in the book. I put in a grand total of 30-36 hours of studying over 4 weeks, including the 11 hours of practice tests.
 
Last edited:
1) 521: 132/129/131/129 on 7/20
2 + 3)Method:
C/P: Read EK, drilled formulas, did only EK discrete questions. Went through AAMC QPs + SB. Had issues with electricity + fluids, so did some UW review of that (from a free trial).
CARS: Khan Academy in Jack Westin format under timed conditions, CARS QP 1
B/B: read EK Bio 1 and Bio 2. Bio 1 is actually quite helpful if less in depth. Revised biochemical pathways, AAs, bases, PPP, TCA etc. from Lehninger, which was the most clear and accurate for me. Did 1 Bio QP and Bio SB
P/S: Crammed 100 page P/S document the 4-5 days before, did P/S SB
4) AAMC Sample: 129-130/132/131/127 (7 days out) and AAMC FL 1 130/130/130/126 (5 days out)
5) Biology, have done a lot of biology research. Graduated by the time I wrote. Honestly felt undergrad was harder than the exam.
6) Unless you are 100% sure you are going to use everything, I'd advise buying practice materials 1 by 1. Bought the whole AAMC pack to not use 2 FLs and 2 QPs. Also did not need the Kaplan set or the TPR books I had. Stick to AAMC material if you review well and aren't utterly perplexed by concepts, 3rd party is strictly for content review and should not be used to extrapolate anything. Take FLs earlier than 7 days out and don't fall into the trap of, "oh everyone takes 10 altius FLs before taking an AAMC test," - know yourself! You went to school for 15-16 years by now, you know your strengths and weaknesses. Cater to those. If you think something will improve your studying, go for it. I knew that reading a physical book and making Anki flashcards from it would be very slow, so I finally found electronic copies of EK. (which I can share) This allowed me to finish content much faster. Take care of your physical, emotional, and mental health. Do not sacrifice sleep for more hours. You should be at your sharpest. Remember to eat (big issue for me when I'm focused). Find a good study space.
7) I don't know how to classify how long - technically, I was messing around with the books from June 1st, 2nd, but I was horribly unproductive and distracted. If I'm being honest, I probably properly started ~June 21st, then took a 4-5 day (unanticipated) break from 7/2 to 7/6, then went HAM 7/7 to 7/19. So ~22 days? The last 12, I did 8-12 hours a day, staying in the library sometimes until 1 or 2 AM.

Schedule: For me, having a day to day schedule never really worked. Like, 1 day - I'll do 1 EK chapter etc. This 'easy' workload, makes me slack off and waste more time. I really focused on goals. Review all the EK stuff. Know Biochem well. Review and drill all the formulas (via Anki). Focus on AAMC material, i.e. SBs and revise carefully. Keep a list of ongoing questions and topics you need to review. The objective >> the method for me. If this was a life/death scenario, would you be satisfied with X hours, or would you want to do everything you could to get the best possible outcome? Obviously it's a bit tamer, but I definitely fall into the 2nd category. Also, don't slack on P/S, it's boring and I happily braindumped all of that information, but it's still an important section. During preparation, I'd also advise reviewing FLs/practice passages carefully and optimizing a strategy for each section. C/P, for ex: I wrote everything out, even when reasoning.

The night before the test, sleep a lot. Take melatonin or something else if needed. During the test, have an aggressive mentality. Scratch out wrong answers, write things down if it helps, flag and come back...don't get overwhelmed. You can do it!
Let me know if you have any other questions. Good luck!
 
  • Like
Reactions: 2 users
Members don't see this ad :)
1) Your individual scores and composite score
Chem/Phys 130, CARS 126, BIO/BIOCHEM 128, PSY/SOC 130

2) The study method used for each section
For the Chem/Phys, Bio/Biochem and Psy/Soc I started by reviewing the content for each section using my preparation books, Kahn academy videos and other Youtube videos. Then I used the AAMC question packs/section banks for practice.

For CARS preparation, I utilized the AAMC CARS practice passages and Testing Solutions practice passages.

3) What materials you used for each section(Kaplan, TPR, Examkrackers, AAMC, TBR, etc)
I used Kaplan prep materials and AAMC practice essentials for each section. Additionally, I used Andrew's resources on MCAT Self Prep, which were tremendously helpful.
I used testing solutions for extra CARS practice.
I used Kahn academy and other online videos for Chem/Phys, Bio/Biochem, and Psy/Soc practice.

4) Which practice tests did you use? (Optional: include scores)
I used Next Step and AAMC practice tests. My first practice test prior to starting to study was a 501. My final AAMC practice test score 1 week before my test was a 512.

5) What was your undergraduate major?
Biology, with a Spanish minor

6) Any other tips you may have for those of us who still have this test lurking over us?
Take breaks. It is more about the quality of your studying than the studying. In my opinion, it is more effective to study diligently for one hour without any distractions (texting, social media, etc.) than to study for three hours and stopping every 15 minutes to message a friend (I speak from experience lol). Also, remember you are so much more than a score. Obviously, the MCAT is a huge deal because it is a way to standardize all applicants, however, at the end of the day, it is just one number. It is easy to get down and depressed while studying for the MCAT when you are unhappy about your performance or start comparing yourself to others. So don't do it. Do the best that you can do! If you are meant to be a doctor, medical schools will see your compassion, work ethic, and servant minded attitude despite your test score.

7) How long did you study for the MCAT?[/QUOTE]
I started studying after graduation so about May 1, 2018 and took my test on August 3, 2018.
 
Alright - I really didn't think I was going to be someone who got to post this, but since my situation might be similar to some others, here ya go:

1) Comp 514 CP 128 CARS 131 BB 126 PS 128

2) The study method used for each section
So this is where it gets a little sticky. I took basically all of my prereqs within the past 15 months, so all my OChem. Bio, BioChem, and Physics was pretty fresh. I'm a non-trad with a pretty broad liberal arts background so the CARS was always my best section. I started preparing for the test on 6/19 and took it on 8/18.

For all of the sections, I read every page of the Kaplan books, did every single practice question in each chapter when it was completed. If the chapter was something I was going over a second time for whatever reason I did all of the concept check questions (section questions). Like actually wrote the answers in the boxes instead of looking at them going "yah I know that crap".

CP- Read the books. For each of the High Value sections, if I hadn't internalized the material I watched youtube/Khan videos for them. I never did any additional problems that weren't in the book. Re-read the material after videos. If I couldn't answer the concept check (end of section) questions for these with relative ease, I revisited the videos and re-read the chapter. I would say this was actually my shakiest material. I just couldn't relate to the physics portions as well as I should have been able to. The chemistry material was well practiced because of my course work. The only thing I had trouble with in chemistry was remembering how quantum numbers worked and balancing redox reactions. I hammered those hard enough to feel like I could make good guesses if I saw a problem. For physics, I made sure to flashcard EVERY formula I didn't already know. I had some real trouble with my physics practice. I basically got lucky that there wasn't a lot of material on my exam I didn't know. I drilled physics formulas constantly for the last 2 weeks. I traveled with notecards. I made sure that any formula on the kaplan quicksheet, I felt comfortable enough to guess with it at least if I couldn't master the concept. This method is what got me the score on CP - I have no doubt.

CARS - Didn't spend any extra time on this. In fact, this is the only book I only read once. I also didn't do all the practice material for this one. I realize this isn't a help for anyone else, but I'm 40 years old with a liberal arts degree and an MBA. I read and write constantly throughout the day and have done so for the past 15 years. I have read so much political theory and social science material that the exam passage topics were almost all familiar. Again, I realize most of you think most of that stuff is obscure, but to me it was as comfy as an old pair of jeans.

BB- Just read the Kaplan book over and over. I didn't have ANY anatomy (brain, sensory, nervous) background. I leaned on my Kaplan material heavily for this. Quicksheets helped focus me on what was expected/proscribed. This is the one place where I branched out to random youtube searches to find lectures and quick reviews on membrane potential, kidney function, nervous system anatomy, lipid metabolism, and immune system. Most of the other topics I kept to the review books and classroom material. There was some significant Ochem in this section for me which oddly enough I was pretty strong in during my coursework. I only read my biochem review book once. I wore the print off the pages off the bio book for the anatomy pieces. I could answer most of the chapter questions and then go back and try them again after reading another few chapters and it be all gone. So I would do it again. Yeah, this is gonna bite me in the ass in med school I know.

PS - This was my weakest practice area. On all my practice material I was getting < 50% right. The Kap books were the only resource I had. I read the book at least 3 times. This is the only section I did practice passages on outside my Full/Half length exams. I wish I had been smart enough to get advice on this from someone on here for this section. I was panicked on test day about this part. I used the kaplan flashcards and the anki flashcards for the Psych/Soc terms more than any other section. Brute force approach. It had been almost 20 years since my psych 101 and sociology 101 classes. Talk about cruft.

3) What materials you used for each section(Kaplan, TPR, Examkrackers, AAMC, TBR, etc)
Kaplan In-Person, Khan Academy (vids only), Awesome Lectures (anatomy discussions), Anki flashcards, Kaplan flashcards, KAPLAN QUICK SHEET!!!

4) Which practice tests did you use? (Optional: include scores)
Kaplan 1/2 length assessment - day 2 of class (500)
Kaplan Full length - Wednesday before test (501)

5) What was your undergraduate major?
BA Government & Politics - Pre-Law
MBA - Finance/Accounting

6) Any other tips you may have for those of us who still have this test lurking over us?
Content was my weak spot. I found that doing practice passages was good for timing, but applying the "critical thinking" aspect of using the passages wasn't something valuable to me. I was too impatient (and too time constrained) to dedicate a lot of time to these. In fact I rushed through my practice exams which probably depressed my score on them. I feel if I had taken more care with these and learned how to pay attention to details in questions better, I could have raised my practice score AND my actual score. If I had more time before my test I would have done this. In retrospect "hurrying" through these practice tests did help to point out where I was actually weakest. If I didn't know an answer I usually just picked something that looked good and moved on and didn't revisit. I did this with the sample passages as well. It was a good strategy for pointing out weaknesses, but not really good for resolving them. I would recommend you do this at least once though if you have the time. Jam through a FL or Half-Length and just see where you are with that type of material. Use the wrong answers to point out where your holes or bad "question recognition" patterns are. Being able to quickly answer correctly is what this test is about. Practice THAT skill.

7) How long did you study for the MCAT?
Well, like I said I took BIO1, BIO2, OHEM1, OCHEM2, PHYS1, PHYS2, and BIOCHEM all within the 13 months preceding the test.
I spent the last two weeks studying about 8-10 hrs a day, every day but the day before test day and the Saturday before that. Prior to that I was in a BioChem class. I would estimate I spent about 30 hours a week on strictly MCAT material for the entire month of July. So that's about 250 hours (including kaplan class time) plus the biochem class I was in.

Since today is the last test day for 2018, good luck to all you 2019'ers!!! Happy to help/answer any questions anyone has. Also a non-trad so maybe I can give some tips there? I doubt it though. :D

-Josh
 
Last edited:
  • Like
Reactions: 1 users
Alright - I really didn't think I was going to be someone who got to post this, but since my situation might be similar to some others, here ya go:

1) Comp 514 CP 128 CARS 131 BB 126 PS 128

2) The study method used for each section
So this is where it gets a little sticky. I took basically all of my prereqs within the past 15 months, so all my OChem. Bio, BioChem, and Physics was pretty fresh. I'm a non-trad with a pretty broad liberal arts background so the CARS was always my best section. I started preparing for the test on 6/19 and took it on 8/18.

For all of the sections, I read every page of the Kaplan books, did every single practice question in each chapter when it was completed. If the chapter was something I was going over a second time for whatever reason I did all of the concept check questions (section questions). Like actually wrote the answers in the boxes instead of looking at them going "yah I know that crap".

CP- Read the books. For each of the High Value sections, if I hadn't internalized the material I watched youtube/Khan videos for them. I never did any additional problems that weren't in the book. Re-read the material after videos. If I couldn't answer the concept check (end of section) questions for these with relative ease, I revisited the videos and re-read the chapter. I would say this was actually my shakiest material. I just couldn't relate to the physics portions as well as I should have been able to. The chemistry material was well practiced because of my course work. The only thing I had trouble with in chemistry was remembering how quantum numbers worked and balancing redox reactions. I hammered those hard enough to feel like I could make good guesses if I saw a problem. For physics, I made sure to flashcard EVERY formula I didn't already know. I had some real trouble with my physics practice. I basically got lucky that there wasn't a lot of material on my exam I didn't know. I drilled physics formulas constantly for the last 2 weeks. I traveled with notecards. I made sure that any formula on the kaplan quicksheet, I felt comfortable enough to guess with it at least if I couldn't master the concept. This method is what got me the score on CP - I have no doubt.

CARS - Didn't spend any extra time on this. In fact, this is the only book I only read once. I also didn't do all the practice material for this one. I realize this isn't a help for anyone else, but I'm 40 years old with a liberal arts degree and an MBA. I read and write constantly throughout the day and have done so for the past 15 years. I have read so much political theory and social science material that the exam passage topics were almost all familiar. Again, I realize most of you think most of that stuff is obscure, but to me it was as comfy as an old pair of jeans.

BB- Just read the Kaplan book over and over. I didn't have ANY anatomy (brain, sensory, nervous) background. I leaned on my Kaplan material heavily for this. Quicksheets helped focus me on what was expected/proscribed. This is the one place where I branched out to random youtube searches to find lectures and quick reviews on membrane potential, kidney function, nervous system anatomy, lipid metabolism, and immune system. Most of the other topics I kept to the review books and classroom material. There was some significant Ochem in this section for me which oddly enough I was pretty strong in during my coursework. I only read my biochem review book once. I wore the print off the pages off the bio book for the anatomy pieces. I could answer most of the chapter questions and then go back and try them again after reading another few chapters and it be all gone. So I would do it again. Yeah, this is gonna bite me in the ass in med school I know.

PS - This was my weakest practice area. On all my practice material I was getting < 50% right. The Kap books were the only resource I had. I read the book at least 3 times. This is the only section I did practice passages on outside my Full/Half length exams. I wish I had been smart enough to get advice on this from someone on here for this section. I was panicked on test day about this part. I used the kaplan flashcards and the anki flashcards for the Psych/Soc terms more than any other section. Brute force approach. It had been almost 20 years since my psych 101 and sociology 101 classes. Talk about cruft.

3) What materials you used for each section(Kaplan, TPR, Examkrackers, AAMC, TBR, etc)
Kaplan In-Person, Khan Academy (vids only), Awesome Lectures (anatomy discussions), Anki flashcards, Kaplan flashcards, KAPLAN QUICK SHEET!!!

4) Which practice tests did you use? (Optional: include scores)
Kaplan 1/2 length assessment - day 2 of class (500)
Kaplan Full length - Wednesday before test (501)

5) What was your undergraduate major?
BA Government & Politics - Pre-Law
MBA - Finance/Accounting

6) Any other tips you may have for those of us who still have this test lurking over us?
Content was my weak spot. I found that doing practice passages was good for timing, but applying the "critical thinking" aspect of using the passages wasn't something valuable to me. I was too impatient (and too time constrained) to dedicate a lot of time to these. In fact I rushed through my practice exams which probably depressed my score on them. I feel if I had taken more care with these and learned how to pay attention to details in questions better, I could have raised my practice score AND my actual score. If I had more time before my test I would have done this. In retrospect "hurrying" through these practice tests did help to point out where I was actually weakest. If I didn't know an answer I usually just picked something that looked good and moved on and didn't revisit. I did this with the sample passages as well. It was a good strategy for pointing out weaknesses, but not really good for resolving them. I would recommend you do this at least once though if you have the time. Jam through a FL or Half-Length and just see where you are with that type of material. Use the wrong answers to point out where your holes or bad "question recognition" patterns are. Being able to quickly answer correctly is what this test is about. Practice THAT skill.

7) How long did you study for the MCAT?
Well, like I said I took BIO1, BIO2, OHEM1, OCHEM2, PHYS1, PHYS2, and BIOCHEM all within the 13 months preceding the test.
I spent the last two weeks studying about 8-10 hrs a day, every day but the day before test day and the Saturday before that. Prior to that I was in a BioChem class. I would estimate I spent about 30 hours a week on strictly MCAT material for the entire month of July. So that's about 250 hours (including kaplan class time) plus the biochem class I was in.

Since today is the last test day for 2018, good luck to all you 2019'ers!!! Happy to help/answer any questions anyone has. Also a non-trad so maybe I can give some tips there? I doubt it though. :D

-Josh
Did you also take the AAMC FLs?
 
1) Chem/Phys - 123; CARS - 122; Bio - 126; Psych - 122; Total of 493.
2) I went over the materials using mostly khan's academy videos and ExamKrackers. I feel they covered almost all of the topics.
3) ExamKrackers for Bio/Biochem and Psych/Sociology; all of AAMC tests.
4) AAMC tests and Kaplan tests - I always scored around 496-500.
5) Undergraduate and master's degrees in Cellular/Molecular Biology
7) Studied the ExamKrackers and Khan's videos for 3 months after work leading up to the test although I had covered most materials prior to that using online sources like mcat-review and other random materials. Did practice tests like AAMC and some Kaplan tests on Saturdays leading up to the actual test.

Here is what I think and need your valuable opinions on it. I feel like I know the materials and just don't know how to work through the passages. That I'm not a native English speaker might play a huge role. So I'm curious as to how I should strategize my study plans. I'm thinking of a paid course in the next three months and study for close to 8 hours a day with no other obligations.
 
  • Like
Reactions: 1 user
1) Chem/Phys - 123; CARS - 122; Bio - 126; Psych - 122; Total of 493.
2) I went over the materials using mostly khan's academy videos and ExamKrackers. I feel they covered almost all of the topics.
3) ExamKrackers for Bio/Biochem and Psych/Sociology; all of AAMC tests.
4) AAMC tests and Kaplan tests - I always scored around 496-500.
5) Undergraduate and master's degrees in Cellular/Molecular Biology
7) Studied the ExamKrackers and Khan's videos for 3 months after work leading up to the test although I had covered most materials prior to that using online sources like mcat-review and other random materials. Did practice tests like AAMC and some Kaplan tests on Saturdays leading up to the actual test.

Here is what I think and need your valuable opinions on it. I feel like I know the materials and just don't know how to work through the passages. That I'm not a native English speaker might play a huge role. So I'm curious as to how I should strategize my study plans. I'm thinking of a paid course in the next three months and study for close to 8 hours a day with no other obligations.
If you're pulling 123 in CP and 122 in PS, then your content is really lacking. You have to really learn and obsess over the material. Once you feel comfortable with the material, go on to do all AAMC question and section banks. Practice applying your knowledge and learning the patterns of how AAMC likes to ask questions. I went from a 125 in BB first time I took the exam to a 131 a year later and I credit this gain to an intense biology post bacc I completed. I honestly think if you become comfortable with all the material, everything else will follow. Also, I sucked at CARS, so I can't help you there.
 
  • Like
Reactions: 1 user
If you're pulling 123 in CP and 122 in PS, then your content is really lacking. You have to really learn and obsess over the material. Once you feel comfortable with the material, go on to do all AAMC question and section banks. Practice applying your knowledge and learning the patterns of how AAMC likes to ask questions. I went from a 125 in BB first time I took the exam to a 131 a year later and I credit this gain to an intense biology post bacc I completed. I honestly think if you become comfortable with all the material, everything else will follow. Also, I sucked at CARS, so I can't help you there.
Thanks for the reply. Do you recommend any particular courses and materials?
 
Thanks for the reply. Do you recommend any particular courses and materials?
I don't think you need to pay for a course. I just used the kaplan 7 book set and the free khan academy videos. Go through the books, take great notes, complete all the practice questions at the end of the chapter and make sure to review the questions after. Really understand why the correct answer is right. Afterward, follow up with Khan videos if you are still unsure. Doing this, it would take me at least 2 hours to go through one chapter. Dedicate at least a full month to doing content.

After you finished going through all the content then you should start the section banks and question banks from AAMC, this is where your notes come in handy. Don't worry about doing the passages timed, instead focus on your content and learning how to apply, use your notes to solve problems before looking at the solution. Later on, you can start incorporating full length exams into your studying weekly. I took about 8 full lengths before I took my real MCAT. Also, use the 100 page Khan academy notes found on reddit for PS, I thought the kaplan PS book was trash.
 
  • Like
Reactions: 1 user
I don't think you need to pay for a course. I just used the kaplan 7 book set and the free khan academy videos. Go through the books, take great notes, complete all the practice questions at the end of the chapter and make sure to review the questions after. Really understand why the correct answer is right. Afterward, follow up with Khan videos if you are still unsure. Doing this, it would take me at least 2 hours to go through one chapter. Dedicate at least a full month to doing content.

After you finished going through all the content then you should start the section banks and question banks from AAMC, this is where your notes come in handy. Don't worry about doing the passages timed, instead focus on your content and learning how to apply, use your notes to solve problems before looking at the solution. Later on, you can start incorporating full length exams into your studying weekly. I took about 8 full lengths before I took my real MCAT. Also, use the 100 page Khan academy notes found on reddit for PS, I thought the kaplan PS book was trash.
Yeah I'm thinking about purchase the Kaplan 7-book and the MCAT 528. I did think about doing the passages without timing myself.
 
Yeah I'm thinking about purchase the Kaplan 7-book and the MCAT 528. I did think about doing the passages without timing myself.

I used the Kaplan 7-book and the MCAT 528 and I thought it really helped me with my preparations. One thing I really liked was that the 7-book system came with 3 full-length practice tests.
 
  • Like
Reactions: 1 user
The course helped me with some anatomy and behavioral stuff I had never seen before. If you've done all your coursework including AP, Biochem, and genetics, then you don't need the in person. There are some "Kaplan methods" for doing CARS, but it's basically an outlining method. I'm sure there are free pointers that are just as good. There's some stuff about "wrong answer pathology" and more detail about the question types, but I don't feel they help you as much as just practicing passages does.

Most of the lecture type material in the Kaplan class is available online for free.

Josh


Sent from my Pixel XL using Tapatalk
 
  • Like
Reactions: 1 user
1) Overall Score: 524 (100th percentile) PS: 132 CARS: 130 BS: 132 Psych: 130

2) Study Method: I reviewed content and took notes on everything I thought was particularly important, which I later transcribed into notecards. I would quiz myself on all of my notecards weekly. I met in a group meeting every 1-2 weeks to do practice question sets, after which we would share our answers and thoughts on each question. This was particularly helpful--not only could we become familiar with the MCAT and how it is written, but we were able to better understand useful strategies and how to attack each type of question with purpose. I formed a group of other students with whom I met weekly and taught the concepts that were most difficult for me, which allowed me greater exposure on my weakest topics. Really, though, it comes down to understanding the exam. The more exposure you get to the exam, the more comfortable you will become. I did a full length exam about once a month until I reached the final month of study, when I took one every 2-3 days.

3) Study Materials: Altius provided me with a large book that covers all the content, as well as an online copy of the same. They also provide 10 full-length practice exams, 30+ mini-exams of a few passages each, and a workbook of hundreds of CAR passages for practice--I also used Jack Westin passages for practicing CAR. They also provide students with the AAMC Official Guide test, Sample Test, and the 3 full-length exams. In short, I used everything they gave me.

4) Practice Tests: I used the online Altius tests that come with their program throughout my study, but saved most of them for my final month of preparation. I also took the 3 practice tests available through the AAMC. I wouldn't recommend waiting to take all the AAMC tests at the end of your study, because then you have no time to study according to what the AAMC puts out.

Altius FL scores:
Exam 1: 127/126/126/126 (505)
Exam 2: 130/126/129/127 (512)
Exam 3: 128/126//131/128 (513)
Exam 4: 129/130/131/128 (518)
Exam 5: 131/128/130/130 (519)
Exam 6: 130/128/130/126 (514)
Exam 7: 130/126/128/126 (510)
Exam 8: 131/129/130/129 (519)
Exam 9: 132/130/131/130 (523)
Exam 10: 129/127/129/126 (511)

AAMC1: 131/130/132/129 (522)
AAMC2: 132/128/132/129 (521)
AAMC3: 132/129/132/130 (523)

5) Undergraduate major: Molecular Biology, minor in Chemistry

6) Tips: Honestly I think the test-prep industry as a whole emphasizes content way more than it needs to be. The MCAT became so much more doable when I started looking at it as a test of how I think and not so much what science facts I know. Obviously you have to be familiar with the science, but understanding the test will do much more in helping you reach your goals. Taking enough practice tests a huge help; don't wait too long to take the AAMC tests!!!! If you wait until the week of the real test then you won't have enough time to fix your mistakes and fortify your weak spots. Also learning to work with the information that was given by the passage is very valuable. If you want to do well, you have to work for it.

7) Time Spent Studying: I studied 5-10 hours/week September-December and kicked it up to 15-20 hours/week January-April. Some of this time was spent reviewing content, and Altius provides short practice question sets that do a decent job of imitating test questions. I also practiced reading CAR passages about 3 hours each week. In May I took a practice test every other day (mostly Altius tests, with a few AAMC tests thrown in) and reviewed the tests on my off days. By the time I took the test in late May, it was just like any other day of studying.
 
  • Like
Reactions: 1 users
1) CP: 131, CAR: 130, BB: 131, Psych/Soc: 130; Composite 522 (99th percentile)

2) I used a combination of personal study using my Altius review book, Anki flashcards that I made (with spaced repetition), group study, personal tutor, and practice tests. I also supplemented with videos from Khan Academy.

3) I used Altius (Gold) program (the above methods are all required by Altius).

4) Which practice tests did you use? I used the 10 Altius practice tests and the 3 AAMC practice tests. The three AAMC ones were all within four points of my actual score, but I took them at the end of my study period. I found the Altius tests gave me a slightly lower score than the AAMC ones, but they felt basically the same so I do recommend them. I can't say about any other companies' materials because I only used the Altius/AAMC.

5) What was your undergraduate major? English and Neuroscience double major, Gerontology minor

6) Any other tips you may have for those of us who still have this test lurking over us?
Honestly, take timing and practice tests seriously, and begin your study early. Make MCAT your priority for as long as you study, and be willing to push back the test a little if you're not prepared.

ALSO: REGISTER EARLY (OCTOBER 25th for 2019) SO YOU CAN GET A SPOT WHERE YOU WANT.

7) How long did you study for the MCAT? I studied about 15-25 hours a week for about 9 months. I struggle with science/math more than the average pre-med, I think, so slightly less could be sufficient for some people. Base it off your needs and your performance on practice tests.
 
Last edited:
I want to start by saying that my plan was the combination of suggestions from SDN and reddit and it worked well but was more work than I thought it would be. Know what you are up against before you start. Be 100% committed to this exam. I took the MCAT prior to taking biochemistry and fortunately had an exam with very little biochemistry. Everyone from roommates to my big sib to a TA warned me against doing this and I got lucky.

1) Comp 516 CP 132 CARS 122 BB 132 PS 130

2) The study method used for each section
I started in December 2017 by researching what people did that worked and didn't work. I scoured reddit and sdn, picked my big sib and her friend's brains about their experience, and exhaustively looked at scores for people here and reddit to see what they used. With my GPA I was hoping to break 510 before I started, so I made a tally sheet where materials used by people scoring where I wanted got +1 and materials used by people below that got a -1.

I borrowed the review books I was considering during winter quarter and used them to supplement classes I was taking. I figured that I'd kill some birds with just one stone studying for my classes and the MCAT. Materials are way more different than I thought they'd be and you really need to find what speaks to you. I eliminated materials for each section one by one until I had what I thought were the best one or two for me.

C/P- Like I mentioned, I had no biochemistry before the MCAT so I needed more detailed books. Or so I thought. It ends up that I was able to pick up much of biochemistry (I'm taking it this quarter and it's a breeze) through doing passages and going over what I didn't know using KA videos. Berkley Review (TBR) is far and away the best materials out there for C/P. I used their organic book, general chemistry book, and physics book. What really helped me were questions built into the reading to test me on concepts and tricks. Then you have three different homework assignments for each chapter with answer explanations that emphasize problem solving. Read every explanation, even if you got a question right. Their shortcuts are pure gold. I went from most scared of this section to it becoming my strongest. Be ready for anything. After I finished my book review about two months before my exam, I went on to AAMC Q packs and then AAMC SBs. This is a must. Don't be discouraged if you get low scores on TBR materials or the AAMC SBs. I was in the 60s and 70s for most of them.

CARS - I honestly have no ^@%$ing clue what to suggest here. Before starting my review I eliminated crappy material and had it down to EK and TPR as my primary sources. I really liked this section at first but never saw improvements as I went along. I did CARS two to three time per week from multiple sources. I followed advice from everywhere I could find it. I honestly think I lost my confidence on practice exams towards the end and went in defeated. My actual CARS section on test day felt kinda easy compared to practice. My CARS score is terrible just like my SAT was. I think some people are made to do well on CARS and other people are me. For the last month I tried CARS sections from different FLs I bought from three different sources and put plenty of time into practice.

B/B- I used many different materials for this section starting with TBR biology books, EK biology books, NS biology books, and KA videos. You have to know your stuff. I quizzed myself every day on the material. I made flash cards for every question I missed. I put 10% of my time into review and 90% into practicing passages and questions. More than the other sections, B/B has crazy data charts and wth questions. FLs from Altius, EK, and TBR helped me prepare for these more than book review. AAMC SBs were the most realistic of everything I used for this section. If I had it to over again I probably just get the TBR book (cuz I needed content for stuff I hadn't seen before) and AAMC SBs and then do the B/B section from every FL I could find. If you know you bio well then EK will suffice for review. I probably wasted ten hours annotating the KA videos I watched. If you use KA videos watch them through without taking notes and do passages right after.

P/S - Get the 100-page document or the 300-page document. My test had so many terms you had to know. I used TPR for this section and it was adequate. I honestly don't think it matters what materials you use here as long as you do the KA passages. Those are by far the closest thing to the MCAT you'll find.

3) What materials you used for each section(Kaplan, TPR, Examkrackers, AAMC, TBR, etc)
C/P- TBR organic book, TBR general chemistry book, TBR physics book, AAMC Q packs, and then AAMC SBs.

CARS - EK 101 and TPR CARS book. For CARS practice after exhausting those books I did CARS sections from FLs other than AAMC.

B/B- TBR biology books. I also looked EK biology books for quick reference and AAMC QPs and SBs.

P/S - The 100-page reddit document, TPR psychology/sociology book, and KA passages.

4) Which practice tests did you use? (Optional: include scores)
Test 1 (early June, before doing much review) AAMC sample: C/P 70%, CARS 68%, B/B 74%, P/S 80%
FL1 (8/1): TBR1 C/P 130, EK1 CARS 123 (ouch), Altius1 B/B 127, EK1 P/S 128 = 508 total
FL2 (8/7): TBR2 C/P 129, EK2 CARS 122 (starting to stress), TBR1 B/B 129, Altius1 P/S 129 = 509 total
FL3 (8/10): Altius1 C/P 131, Altius1 CARS 125, EK1 B/B 127, TBR1 P/S 128 = 511 total
FL4 (8/15): EK1 C/P 129, EK3 CARS 124, Altius2 B/B 128, EK2 P/S 129 = 510 total
FL5 (8/20): TBR3 C/P 131, Altius2 CARS 126, TBR2 B/B 129, Altius2 P/S 128 = 514 total
FL6 (8/23): AAMC 1 C/P 131, CARS 126, B/B 131, P/S 130 = 518 total
FL7 (8/30): AAMC 2 C/P 131, CARS 124, B/B 131, P/S 129 = 515 total
FL8 (9/5): EK3 C/P 127, Altius3 CARS 124, TBR3 B/B 129, Altius3 P/S 128 = 508 total
FL9 (9/13): AAMC 3 C/P 131, CARS 123, B/B 130, P/S 128 = 512 total

5) What was your undergraduate major?
Molecular Biology

6) Any other tips you may have for those of us who still have this test lurking over us?
Take a realistic FL early to get a feel for the exam's way of presenting material and asking question. This is big and the sooner the you know how they ask things the better you will study. I loved that I mixed up my FLs. It gave me a feel different writing styles from section to section which is EXACTLY what you will feel on MCAT day. Set a schedule and stick to it. Give yourself five things to do everyday, big or small, and check them off as you complete each. It's a really great feeling and highly motivation to check things off as you make progress. Get a study spot and turn of your cell phone when you are there. Be you! Don't try every half-baked suggestion people give you while you're studying. No disrespect intended here, but the MCAT now is nothing like the one the GP in the lunch room took thirty years ago.

7) How long did you study for the MCAT?
I started skimming in December 2017 and used review books as references while taking classes. I had the TBR books available when I took physics and o chem. I didn't start hardcore mode until after finals and then studied for three months. Overall I probably studied 800 hours total.
 
1) 130/128/130/129 -> 517
2) I used Altius' study method for each section. This entailed using their workbook and learning to conceptualize and apply concepts.
3) I used Altius materials for every section and supplemented PS with Khan Academy. I also listened to The MCAT Podcast (co-hosted by Next Step) in the car and while walking around campus.
4) I used Altius practice FLE's and the AAMC practice tests. My exam scores fluctuated a lot, but overall, they were pretty accurate of what I got on the real test.
5) Biology
6) Give yourself plenty of time to study, ditch extracurriculars that are sucking up your time, sleep 8 hours per night, see your friends/family, and exercise. These elements are overlooked but crucial for physical/mental health and stamina that the MCAT requires. Also, be honest with yourself regarding your study habits. If you need to invest in a prep course because you are not good at self-studying, the dollars you drop now to get a good score and get into medical school will pale in comparison to the money you will be making in the future. MCAT prep is an investment of time, energy, and dollars if necessary.
7) I studied for about 6 months
 
  • Like
Reactions: 1 user
1) Your individual scores and composite score
513, forgot composite scores but all good..
2) The study method used for each section
Used Kaplan self paced, class was pointless but books were great review plus the Q bank and FL's also did AAMC FL's

5) What was your undergraduate major?
bui
6) Any other tips you may have for those of us who still have this test lurking over us?
It's all about doing the pre-reqs and doing well in them then studying and putting the time in (3ish months). Study until you get the FL scores you want then study some more.
7) How long did you study for the MCAT?
3-4 months
 
2 Time Retaker.. feel free to ask me anything

1) 503-> 501 -> 512 (129,125,129,129)

So the first two times I used Kaplan books / multiple practice tests/ khan academy videos with an extensive studying plan (40+ hours/ week for ~10 weeks) second time I upped intensity and duration

3rd time I studied 12 hours a day for 28 days using mcatprep528.com (CurveSetter). This was a game changer.

For those who wish to see my study plan, message me and I will send it via email.

Remaining will be regarding latest test.


2) For C/P, B/B, and P/S Reviewed each CurveSetter video in order of most struggled section to easiest. I dedicated each day to a particular topic, reviewed set amount of videos, did a specific amount of practice problems and MOST IMPORTANTLY: reviewed every set of notes every 1 and 7 days following that lecture/ day. This spaced repetition really consolidated the info. Also every pathway I though was important, I wrote it out and stuck it to my wall (wall was and still is covered).

For P/S I made flash cards for each key term I didn't know well and reviewed these frequently. Since most of this section is term based, this helped interweave concepts.

For CARS, I tried to do about 1-2 passages a day, but with only 28 days I focused on sciences


3) What materials you used for each section? mcatprep528.com gave me all I really needed in a clear, but condensed version. I would supplement with khan academy for unclear topics. Felt that Kaplan books for too detailed for efficient review.

AAMC everything - bought it, used it as much as I could.



4) Which practice tests did you use? Have taken 10+.. stuck to AAMC, Next Step, TPR, there are all decently similar. Recent test only took sample AAMC test since I had not taken it previously.

5) What was your undergraduate major?

Neuroscience

6) Any other tips you may have for those of us who still have this test lurking over us?

Be humble, but confident. Learned this the hard way, but definitely a great thing to learn as so many premeds can be arrogant. It doesn't matter how many times you are published or how great your GPA is, you do not deserve anything you don't work for and you're not above or below anyone else based on a score.

That being said, as cheesy as this sounds, if you want something, go take it. It's that simple. Taking the MCAT 3 times sucked and tbh killed my confidence at times, but I sucked it up and kept going because this is my dream and nothing is going to stop me from achieving my dream. My PCP told me if you want it bad enough, keep taking it. Anyone can do this if they put their mind into it.

(Preach over) also take care of yourself physically and mentally (positivity is key). This test is very tough, you know that, I know that, everyone knows that. I wish anyone that reads this the best of luck with your efforts and have some confidence (fake it if you have to).

7) How long did you study for the MCAT?
1st: 3 mos
2nd: 2 mos full time
3rd: 28 days, 12 hr q24 (must be cool if I use this terminology right?)
 
  • Like
Reactions: 3 users
I am currently taking my notes on the most important MCAT topics and making them into podcast episodes. When I was studying I always wanted something to listen to while I was driving/doing lab work, so I decided to make a podcast (plus I've always been a big podcast listener and thought it'd be fun). There is a link at the bottom to the podcast. Its on iTunes, Spotify, and Stitcher and is called "Sam's MCAT Basics". Check it out if you're interested. I will be trying to put out 1 episode a week

1) Your individual scores and composite score:
Comp 514 CP 130 CARS 126 BB 130 PS 128

2) The study method used for each section
For content review: I scheduled all my studying before I started. I took a week to skim all of the Kaplan books to get a good idea of which topics I would really need to hit. Typically I would study one topic in the morning, then switch to another topic in the afternoon. Spent about a month just doing content review.
For practice tests: After finishing the content review I started taking a practice test a week. I would review the practice test the day after I took it and make a list of the topics I missed, then I would review those in as much detail as possible. I did this for about 2 months. I threw in the AAMC Question Packs during this time.

CP:
I skimmed the Kaplan books and wrote down a list of all the topics that I did not feel comfortable in. Then I spent a few weeks doing content review. Kahn videos/ other youtube lessons were helpful for the physics topics I did not understand.
CARS: This was always my worst topic. Lots of practice was really the only way I got better. I mainly used the Official AAMC CARS questions packs, I thought they were most representative. I also did a Jack Weston passage each morning. I was pretty disappointed with my cars score. I had been scoring 127-129 on all my practice tests. I hit cars really hard the last two weeks of studying; I was doing 3-4 hours of timed passages a day. It seemed like it was helping, but after receiving my score back I'm not quite sure it did. In retrospect I would have done more CARS practice.
BB: Spent about a month doing content review here. Supplemented the Kaplan books with Khan/youtube videos for the subjects that I really didn't understand.
Psych: Went through all the Khan Academy videos, took notes and made flash cards. I skimmed the Kaplan book, but found the videos much more helpful.

3) What materials you used for each section(Kaplan, TPR, Examkrackers, AAMC, TBR, etc):
CP: Kaplan, Khan videos, AAMC Question Packs
CARS: AAMC Question Packs, Jack Weston
BB: Kaplan, Khan Videos, AAMC Question Packs
Psych: Khan Videos, AAMC Question Packs
If the Khan videos didn't explain a topic well enough (very rare), I would look up other youtube videos.

4) Which practice tests did you use? (Optional: include scores):
AAMC FL1, 2, and 3: 511, 513, 518
NS 1, 2, 3, 4, 5, 6: 509, 511, 510, 512, 514, 515
AAMC Question Packs:
Official MCAT Biology Question Pack 1
Official MCAT Chemistry Question Pack
Official MCAT Physics Question Pack
Official MCAT Biology Question Pack 2
Official MCAT Critical Analysis and Reasoning Question Pack Volume 1
MCAT AAMC Official Guide Questions

5) What was your undergraduate major?
Chemical Engineering

6) Any other tips you may have for those of us who still have this test lurking over us?
One thing that helped me get through some of the long, boring passages that show up on the MCAT was to read random journal articles each day. I'd spend about 30min-1hr after lunch reading a random paper. Then I would immediately write down everything I could remember and the important take aways of the paper. This especially helped with the P/S section.
Take lots of practice test. Simulate testing conditions.
Make sure you still live your life. Its easy to get really focused on the MCAT and try to study every minute you're free. This is a very fast way to burn out. Also, it would make me study harder during the day if I knew I had something fun happening that night/later that day.
Anki flash cards are the best, used them a ton.
Never even opened the organic chemistry review book. Organic chemistry questions are so low yield.
I thought the NS practice tests were pretty close to the real thing. Although their CARS section wasn't very good.


7) How long did you study for the MCAT?
3 Months, I worked about 20-30 hours a week and tried to study 30-40hours.

Sam's MCAT Basics by Sam Smith on Apple Podcasts
 
  • Like
Reactions: 4 users
I have all the books for the princeton review course, for all subjects. How should I create my study plan for through out the semester. I have 3 months of the semester plus 1 month in the summer. How should I split up the subjects, etc.
 
Hi everyone! I have a question regarding when to take the MCAT in terms of when my prep course ends. I signed up for a Kaplan prep course and was planning on taking the MCAT the day after my final class. But then I was thinking, "what if I learn something in this class that tells me I need more time to study?" I couldn't find a good answer on Google, so I thought to ask here if anybody had a similar experience, or has any useful advice? Should I give myself a week after my final prep class to take the exam??

Personally... I would wait at least 2-3 weeks after a test prep course before taking the MCAT. There is definitely going to be a period where you want "personal prep time" outside of Kaplan. Kaplan is a course that teaches a rigid study method to everyone who takes it. In reality... most students are stronger/weaker in certain areas on the test and require more studying. If it were me, I would take a month after the Kaplan course to adequately address my personal weaknesses.
 
  • Like
Reactions: 1 user
1) Your individual scores and composite score
Overall: 522 (99th percentile), C/P:131 CARS:128 B/B: 132 P/S:131

2) The study method used for each section
C/P:
Going into the test I was very nervous for physics, so I made a formula sheet for every possible physics formula that could be tested as well as all of the units for each variable (this is very helpful for when don’t know the formula, because simple dimensional analysis can get you to the correct answer!) I looked at this sheet every other night before bed and rewrote it a few times from memory. I also reviewed physics content through NS test prep books and videos. Anytime I came across something I did not know on a problem, even if it was simply what an answer choice meant, I reviewed and made a note of it. For chemistry I relied heavily on review books and videos to clarify any topics I was not 100% confident on. The chemistry is a lot of memorization so I recommend using any study technique that works best for you to remember the concepts for test day. For me, this meant writing it on paper several times.

CARS: CARS was easily my biggest fear going into the test and I am very happy I pulled off the score I did. I used Testing Solutions strategy and schedule and think this was crucial for me feeling prepared for test day. This schedule is brutal but it builds stamina and makes you feel as prepared as possible for getting through those 90 minutes during the real thing. I think you should use the other companies tests for practice with stamina and timing, and only analyze the explanations in depth for AAMC question packs since these are the people that write the MCAT.

B/B: I used review books for concepts I needed to brush up on but the biggest factor for my success in this section was a strong biochemistry foundation and doing research. Even though you might not need to know the specifics for topics described in passages (such as mouse modeling, secondary messenger pathways, or cell culture), having a background in how these things work and are applied made me feel more confident reading the passages so I can focus on the answer choices. I suggest reading peer-reviewed journal articles (search Pubmed for interesting topics to you) if you don’t have a background in research, because this section seems to be moving away from heavy memorization and more into interpreting research results and figures. The section bank replicated the test the best, so going through that slowly and understanding the logic behind every question is crucial for this section.

P/S: Anki, anki, anki!! About 4 months out from my test day I made a flashcard for every single concept, definition, topic, question I encountered in TPR Psych and sociology review book. I set the maximum to 20 days so I saw each flashcard several times before my test. Then I made more flash cards for all of the AAMC definitions and questions I encountered in the psych section bank. I even made flashcards for definitions that were in the answer choices but were not a correct answer that I had not encountered before. This was done about a month in advance and I set it so I would see these cards every 2-6 days max, so I saw them several times before the real test.

3) What materials you used for each section(Kaplan, TPR, Examkrackers, AAMC, TBR, etc)
C/P: NS review books, NS review videos, AAMC section banks and question packs for most representative questions.

CARS: TPR hyperlearning book, Examcrackers, Testing solutions, AAMC question packs. Anything I could get my hands on, I did at least 15 full length CARS sections (not including full length tests) before my test and most other days did plenty of passages as well.

B/B: NS review books and videos, AAMC question packs and section banks

P/S: TPR and AAMC section banks

4) Which practice tests did you use? (Optional: include scores)
My plan going into the test was to take over 12 practice tests and do one every Sunday leading up to the test, but once I took the AAMC FL1 I realized the other test companies were a waste of my time because the content was not representative of the real thing and the score deflation didn’t help my confidence. Other test companies should be used for pacing and practicing stamina early on but don’t use those scores to predict how you will do.

1. NS FL1: 512 (128/126/129/129)
2. NS FL2: 510 (128/127/128/127)
3. NS FL3: 510 (128/126/128/128)
4. NS FL6: 511 (128/126/130/127)
5. AAMC FL 1: 517 (130/128/130/129)
6. AAMC sample test: 87% (86/81/93/85)
7. AAMC FL2: 517 (130/127/131/129)
8. AAMC FL3: 517 (131/126/132/128)

5) What was your undergraduate major?
Biology

6) Any other tips you may have for those of us who still have this test lurking over us?
This test is a beast, so do not expect to score high without putting in the effort that such a score requires. If you put in the work and take it when you feel ready (i.e. you’ve studied each concept for long enough to have it locked into your memory and not feel iffy on anything) you can achieve the score you want. I was registered for this test in early 2018 but canceled a few weeks before, which ended up pushing back my entire schedule so that I would be 100% ready when I took it, but it paid off .
Another tip is to focus on AAMC material 1 month before your test and review each question many many times. I tried to get into the minds of the test writers this last month by reading their explanations over and over again, and looking at explanations on reddit when the test makers were vague (tons of written explanations on that site!) Review every single question, even the ones you know well, because sometimes they have “notes” within their explanation that can be applied to another problem or topic.

7) How long did you study for the MCAT?
3 months a few years ago, 6 months while working full-time

Feel free to message me with any questions or specifics about my strategy, good luck!!
 
  • Like
Reactions: 4 users
1) Your individual scores and composite score
523 (129,131,132,131)

2) The study method used for each section
I approached each section essentially the same. I went through about 3 chapters of the Kaplan books each day over the summer and watched Khan academy videos to shore up any weak points. After I finished with content, I started taking practice exams, even though there were still topics that I was not entirely confident on. During fall semester, I would take a practice exam every weekend (except for extremely busy weeks) and review the exam for 2 hours each weeknight. I reviewed slowly, making sure to go over every question multiple times and googling any unknown terms/concepts. Each exam had a google spreadsheet with the incorrect question, why I got it wrong, and relevant content notes. A month before the exam, I began to take a practice exam every 3/4 days and also began doing AAMC practice sets. I began a study guide that I made sure to memorize.

3) What materials you used for each section(Kaplan, TPR, Examkrackers, AAMC, TBR, etc)
I took a lot of Kaplan pratice exams, all of AAMC pratice exams, one Examkracker, four Next Step, and 1 Atilus. Khan Academy is a savior. So is ilectureonline for physics.

4) Which practice tests did you use? (Optional: include scores)
Kaplan: 506, 511, 506, 509, 510, 515
AAMC: 515, 517, 517
Next Step: 512, 511, 510, 514
Atilus (half exam): 506

5) What was your undergraduate major?
Biology and History (the latter helped out with cars)

6) Any other tips you may have for those of us who still have this test lurking over us?
The MCAT is not just a content test. It's impossible to do well without practice, practice, and more practice. Be sure to review the questions you got correct as well because there might be parts that you didn't know or you were just lucky. It's ok to feel stupid while studying; the point of studying is to admit what you don't know and prevent yourself from feeling stupid during the test itself. I had to relearn lens like 10 times and honestly still can't recall the topic that well. Also, know your amino acids. Breathe; its going to be ok as long as you stay consistent and put in the work.

7) How long did you study for the MCAT?
From summer to when I took the exam in mid January. Prob averaged 20 hours a week (higher during summer and winter break).
 
  • Like
Reactions: 5 users
Any retakers willing to give me some advice? I'm looking to retake for 3rd time and want to make sure I do really well this time
 
1) Your individual scores and composite score
526, 132/131/132/131

2) The study method used for each section
With the exception of CARS, my study method was largely identical for each section. I studied for 13 weeks total while working full time 60+ hours a week. I spent my first 3 weeks just reading through the Kaplan books. I didn't make any flashcards, or notes because the point was to get a broad overview of every topic that was fair game. Afterward, I started doing timed practice problems from good sources (AAMC question packs, AAMC section Banks, NextStep question banks, and Uworld were what I used). I did 30 questions at a time (~half of a section's worth of questions) and then reviewed every single question thoroughly. I made Anki cards based on anything I missed, or anytime I encountered a topic that I felt even a tiny bit fuzzy about and reviewed the ever-increasing Anki deck throughout my study schedule. To review the 30 questions in depth took 2-4 hours depending on how I did on those questions. For CARS, I did 1-2 timed passages every study day (I gave myself one day off each week) back-to-back and then reviewed. I also took 10 total timed full-length tests, one each week after my initial 3-week content review.

3) What materials you used for each section(Kaplan, TPR, Examkrackers, AAMC, TBR, etc)
I used Kaplan books. As well as most of the official AAMC material.

4) Which practice tests did you use? (Optional: include scores)
In order: AAMC Sample (~high 80% low 90% in each section), NS 1-5 (~512-518), AAMC #1 (523), AAMC Sample (high 90s in all sections), AAMC #2 (528), AAMC #3 (525)

5) What was your undergraduate major?
Computer Engineering w/ Electrical Engineering minor (CE is often known as Electrical Engineering/Computer Science), but I graduated a long time ago.

6) Any other tips you may have for those of us who still have this test lurking over us?
Stop doing content review and start doing high-quality practice problems. Trust the process and remember, everyone else is just as panicked as you are.

7) How long did you study for the MCAT?
~350-400 hours over 13 weeks, I work full time and have a lot of extracurriculars on my plate so I had to be really efficient with my time. When I sat down to study, I really studied. I did 3-4 hours (one of the question blocks I mentioned above), my 1-2 CARS questions and reviewed my Anki cards every weekday before/after work depending on the day. Took Saturday off completely and then spent 12 hours on Sunday doing a full length and reviewing it.

Good luck everyone, hang in there! :)
 
Last edited:
  • Like
Reactions: 4 users
1) Your individual scores and composite score
526, 132/131/132/131

2) The study method used for each section
With the exception of CARS, my study method was largely identical for each section. I studied for 13 weeks total while working full time 60+ hours a week. I spent my first 3 weeks just reading through the Kaplan books. I didn't make any flashcards, or notes because the point was to get a broad overview of every topic that was fair game. Afterward, I started doing timed practice problems from good sources (AAMC question packs, AAMC section Banks, NextStep question banks, and Uworld were what I used). I did 30 questions at a time (~half of a section's worth of questions) and then reviewed every single question thoroughly. I made Anki cards based on anything I missed, or anytime I encountered a topic that I felt even a tiny bit fuzzy about and reviewed the ever-increasing Anki deck throughout my study schedule. To review the 30 questions in depth took 2-4 hours depending on how I did on those questions. For CARS, I did 1-2 timed passages every study day (I gave myself one day off each week) back-to-back and then reviewed. I also took 10 total timed full-length tests, one each week after my initial 3-week content review.

3) What materials you used for each section(Kaplan, TPR, Examkrackers, AAMC, TBR, etc)
I used Kaplan books. As well as most of the official AAMC material.

4) Which practice tests did you use? (Optional: include scores)
In order: AAMC Sample (~high 80% low 90% in each section), NS 1-5 (~512-518), AAMC #1 (523), AAMC Sample (high 90s in all sections), AAMC #2 (528), AAMC #3 (525)

5) What was your undergraduate major?
Computer Engineering w/ Electrical Engineering minor (CE is often known as Electrical Engineering/Computer Science), but I graduated a long time ago.

6) Any other tips you may have for those of us who still have this test lurking over us?
Stop doing content review and start doing high-quality practice problems. Trust the process and remember, everyone else is just as panicked as you are.

7) How long did you study for the MCAT?
~350-400 hours over 13 weeks, I work full time and have a lot of extracurriculars on my plate so I had to be really efficient with my time. When I sat down to study, I really studied. I did 3-4 hours (one of the question blocks I mentioned above), my 1-2 CARS questions and reviewed my Anki cards every weekday before/after work depending on the day. Took Saturday off completely and then spent 12 hours on Sunday doing a full length and reviewing it.

Good luck everyone, hang in there! :)

Wow that's amazing, congratulations!!! I was wondering if you would be willing to share your Anki deck; I'm still getting the hang of using flashcards and would love to check out different formats. Also, did you consider TPR? If you did, would you mind sharing how you ultimately chose Kaplan? I'm trying to decide right now haha, thanks!
 
1) Your individual scores and composite score

521, 131/126/132/132

2) The study method used for each section

I started about 3 months out, and knew that taking notes would take too much time, and I wouldn't revisit them. For all of the sections (except CARS) I would read through a few chapters of the book and make note cards. The next day I would read the next few chapters, make note cards, then reread the material I read the previous day and do the note cards for the chapter (and any due note cards). Rinse and repeat until the textbook is done. It took a lot of time, but I didn't have to revisit nearly the amount of material that I thought I would. This took about 1.5 months, and then I grinded UWorld/AAMC material for the remaining 1.5 months (I was lucky enough to get the 3 month free trial of UWorld when it was still available). For CARS I did 2 JW passages a day throughout studying (but got lazy towards the middle, and my score reflects that)

3) What materials you used for each section(Kaplan, TPR, Examkrackers, AAMC, TBR, etc)

I used second hand books for everything - between 2/3 years old, but still for the new MCAT that I found on Craigslist/from friends who already took the exam.
Chem: I used EK. I tried to read the EK and Kaplan set, but Kaplan went too in depth for my taste. There's already a crazy amount of information required on the MCAT, and I felt like Kaplan made it more difficult to learn the important stuff. Especially with OChem and Gen Chem, just stick to EK.
Bio/Biochem: I used EK again for Bio, and thought it was great. I would make flashcards on anything I saw on UWorld that wasn't in EK, which was a fair amount, but UWorld B/B was tough. Biochem I actually liked Kaplan, as it went in depth into all the metabolic pathways (even though they gave us the whole TCA cycle on the exam LOL).
P/S: I watched the KA videos at 2x, and after I finished a couple sections I would read the 300 page doc and then do Cubene deck the next day.
Anything I wasn't familiar with at the end of 1.5 months I would either watch AK or KA lectures on and do practice problems. I also listened to various KA lectures when driving.

4) Which practice tests did you use? (Optional: include scores)

A friend let me use his Altius tests, and I took 3 of them sporadically during the first 1.5 months, scoring between 507-515. I thought the passages were very good, but the questions were iffy at times (I stopped taking them when a question asked what kind of secondary structure is prominent in spider's silk without mentioning it in the passage - required background information they said lmao). UWorld is amazing as everyone has already said - just do all of them and read the explainations. I think that if you only do UWorld and understand every question your score will bump up 4-8 points. Then I did all the AAMC material within the last 3 weeks - scored 519/521/520 on all the full lengths, so they were spot on for me. For all of the full lengths I woke up at 6am, drank a couple cups of coffee, and started the exam at 8. Even though it seems tedious, I definitely felt more comfortable during the real deal, as I usually don't start the day until about noon.

5) What was your undergraduate major?

Bio

6) Any other tips you may have for those of us who still have this test lurking over us?

You need to be a jack of all trades and a master of none. If you wouldn't be comfortable answering a question about something, you need to review it until you understand it., but don't waste time learning material you already know. Use Anki - I wish I had during my undergrad. There are plenty of pre-made decks available, and I would be willing to share the decks I made if anyone is interested.

7) How long did you study for the MCAT?

3 months.
 
  • Like
Reactions: 1 users
Got my score this morning! So glad to be done with this monster.

1) Your individual scores and composite score

  • 516 (129/129/128/130)

2) The study method used for each section

  • I was working full-time while studying so I studied about 700 hours over a period of 9 months. I used MCAT Self Prep for my study plan and I would recommend this to anyone. It was very easy to keep to a plan with it and it would have been easy to integrate 3rd party resources had I done that. The biggest benefit to MCAT Self Prep, IMHO is that there is an AAMC mini exam at the end of every section that is basically a 3rd length MCAT using AAMC questions.
  • I did content review for the first 7 months and then took 1 FL every Saturday for the last 7 weeks.
  • C/P - this section required the least work for me because I was a mechanical engineering major and I didn't have to do much with physics. I had just done a post bacc and taken ochem so that was pretty fresh as well.
  • CARS - this section gave me issues for a while, I finally started doing well on CARS when I learned how to avoid the traps that the AAMC always uses.
  • B/B - this section just required reading a lot of passages and learning the best way to dissect them.
  • P/S - 86 page doc and UWorld that I borrowed from a friend.

3) What materials you used for each section(Kaplan, TPR, Examkrackers, AAMC, TBR, etc)
  • I hate textbooks. If I can avoid textbooks then I will. Luckily, there are so many video resources for nearly everything on the MCAT. If you are a visual learner, don't waste any more time on books than you have to.
  • C/P - this was solely KA and AAMC
  • CARS - I did a little Jack Westin, UWorld, Testing Solutions, but none compare to AAMC. I went through both Qpacks 2 times and this was the biggest help. I just dissected why each answer was correct and why each answer was wrong and tried to find patterns. When I figured out these patterns my score in CARS improved significantly.
  • B/B - KA was good for this too, but the SB was really the most important. I went through the SB once while doing my mini exams and then again in the last month before the exam. It was amazing how much better I was able to understand after simply going through a lot of Bio passages over the last few months. This just takes practice!
4) Which practice tests did you use? (Optional: include scores)
Practice Exam or Practice Problem Set NameDateC/P Percent CorrectC/P Predicted MCAT ScoreCARS Percent CorrectCARS Predicted MCAT ScoreB/B Percent CorrectB/B Predicted MCAT ScoreP/S Percent CorrectP/S Predicted MCAT ScorePredicted MCAT Score
NextStep Diagnostic8/21/201863.33%125.056.00%123.054.84%124.076.67%127.0499.0
AAMC Mini Exam 19/20/201863.15%124.747.61%120.780.00%127.325.00%118.0490.7
AAMC Mini Exam 211/14/201889.00%129.070.00%124.760.00%123.364.00%123.8500.8
AAMC Mini Exam 31/5/201989.00%129.089.47%129.588.00%129.082.00%127.0514.5
AAMC Mini Exam 41/23/201973.00%126.385.00%128.085.00%128.565.00%124.0506.8
AAMC Mini Exam 52/6/201977.00%126.870.00%124.780%127.350.00%121.5500.3
AAMC Mini Exam 62/19/201982.00%127.790.00%129.5100.00%132.050.00%121.5510.7
AAMC Mini Exam 72/28/201983.00%127.780%126.785.00%128.560.00%123.3506.1
AAMC Mini Exam 83/8/201986.00%128.3100.00%132.075%126.371.00%125.0511.7
Official Guide Questions3/9/201970.00%125.890.00%129.563.00%124.080.00%126.8506.0
AAMC FL 13/16/2019128.0126.0127.0126.0507.0
AAMC Mini Exam 93/20/201980.00%127.391%130.090%129.775.00%126.0513.0
AAMC Mini Exam 103/21/201982.00%127.785.00%128.090.00%129.783.33%127.5512.8
NextStep FL 13/23/2019129.0126.0127.0126.0508.0
AAMC Mini Exam 113/27/201955.00%123.475.00%125.585.00%128.585.00%128.0505.4
Kaplan FL 13/30/2019125.0126.0127.0125.0503.0
AAMC Sample4/6/201977.97%126.879.25%126.377.97%126.786.44%128.3508.1
Kaplan FL 24/13/2019128.0127.0130.0127.0512.0
AAMC Mini Exam 124/17/2019100.00%132.080.00%126.790.00%129.70.8333333333127.5515.8
AAMC FL 24/20/2019130.0129.0128.0128.0515.0
AAMC FL 35/4/2019129.0128.0127.0129.0513.0


5) What was your undergraduate major?
  • Mechanical Engineering
6) Any other tips you may have for those of us who still have this test lurking over us?
  • Please please please focus on AAMC materials. Nothing else comes close. Go through every single problem that the AAMC offers and then go through them again.
  • I recommend MCAT Self-Prep. It worked really well for me. Don't buy the $1000 plan though. It's not really worth the money. Get the $10 plan and get some of the mastery courses. If you have specific questions on them, ask.
  • UWorld is awesome for P/S
  • If you don't like textbooks like me, don't think that you are doomed to a low score. Use the method that works best for you. There are so many great video resources on the internet these days.
7) How long did you study for the MCAT?
  • 700 hours over 9 months
 
Last edited:
  • Like
Reactions: 1 users
1) Your individual scores and composite score:

Total:
512
C/P: 127
CARS: 128
B/B: 130
P/S: 127

Exam date: 5/18/2019

2) How I studied

So, I work full time at a mentally demanding job doing research. I work anywhere from 40-70 hours/week. I would work out at 4 am during the week and study as soon as I got home, Mon - Thurs. Friday I took off to spend time with my GF (who is also studying the MCAT) to not talk about/look at MCAT stuff. It was a much-needed break from studying. Then, I would spend 7+ hours/day every weekend practicing, studying, and (re)reading chapters/watching KA videos on weaker subjects. I began studying in December off and on; then I got very focused beginning of January.

Chem/Phys: This was my weakest subject by far. When I took a half-length Kaplan diagnostic, I made a 492 composite with a 121 on C/P. I focused on REALLY learning the material by reading through Kaplan books. Once I finished those, I would practice every day. I started with AAMC materials then moved to UWorld. I didn't use up all of my QPacks from AAMC but I used up UWorld for everything but Physics. If my day was set for Gen Chem, OChem, etc. I would double my practice and watch all KA videos on the subject if I didn't 100% understand it. My C/P score jumped from a 121 to a 130 on AAMC FL practice exams. A little bummed I couldn't reproduce that come test day; however, I'm happy with my score nonetheless.

CARS: This one was tough for me. I started practicing with Jack Weston free daily passages after trying out Kaplan's method for tackling CARS passages. Every day, I would do 3+ passages, first thing, until I was comfortable with reading to understand CARS. After a while, the *note taking* part of the Kaplan method became second nature to me. I would be able to sum up what each paragraph had talked about in my head, rather than on paper (which saved me time to answer questions.) This was up and down for me because, some days, I'd get 90% correct on AAMC QPack 1 and/or 2. Other days, I would sit at ~55% correct. Eventually, I would complete AAMC CARS QP2 twice and QP1 once. I also trucked through UWorld CARS questions quickly. One thing I also did was read a book for fun each day for 30 minutes to an hour. This got my brain into reading mode before I practiced CARS.

The best advice I was ever given for this section was: you already know how to read. Just pay attention to everything said and 2/3 of the battle is already done (Comprehension and RWT questions.) Now, practice RBT questions and you're golden. This advice carried me to test day. My FL practice scores ranged from 122-129 with no clear trend. So a 128 on the real deal is phenomenal for me. If you're to take away anything from this, it is: PRACTICE THIS EVERY DAY. I didn't read for fun or anything as a kid; also, my ADHD gave me horrible habits - from skimming around when I would get bored to being easily distracted in general. In college, I read only enough to get A's on my papers. I was scoring 1/6 to 2/7 per passage. Hounding on CARS each day while learning why and how I missed each question brought me from 2/7 to 6/7 per passage. A 128 on my exam was 90th %ile. So if I could get that, so can you.

Bio/Biochem:
To be honest, this section was always easy for me. I got a B.S. in Biology with making a 3.7+ GPA taking nothing but bio/chem classes my junior and senior year. On top of that, I did research as a tech for 2.5 years in undergrad and have been at my research job for almost an additional 2. I read scientific journals almost each week and attend plenty of research lectures/talks a few times per month. I feel like my education always trained me for this section. There were definitely some content gaps for me; but for the most part, I didn't study this one too much. My lowest score on FL practice tests was a 127, and my highest was a 131. If I could go back, however, I would definitely study more of lab techniques and more micro (genetics, prokaryotic biology, etc.) That stuff is a big part of the MCAT. I also wish I utilized more of my materials here too. It wasn't because I thought I was "too good" for that. However, I prioritized my weak areas (C/P and CARS) and just left B/B for any time left before bed (unless I had B/B assigned to practice/study that day.)

Psych/Soc: This section was hard for me, despite being 2 classes away from having a major in psychology. What I should have done was went through premed95's deck and memorize the terms. Kaplan was not a big help. It taught it conceptually (which would have been fine if P/S wasn't just a term test.) I would have probably gotten a 130-132 on this section if I just spent the time to read and memorize the terms. What I actually did was read the Kaplan book and watched KA videos over and over. With a ton of practice from AAMC and QWorld. My friend went through premed95's deck and made a 132 on P/S. She memorized the deck, quizzed herself twice on all the terms, and then went into the test. That was it (other than reading through the EK book a little.)

EDIT: I would like to add that you should ALWAYS review research methods, design, and ethical dilemmas. Having taken it twice (once two years ago,) the AAMC seems to be adding more research methods to their practice materials and FLs (refuse to comment what my exam had.) I had done research for almost 4 years, combined. It came second nature, but there have been plenty of questions that stumped me. DO NOT OVERLOOK IT. You won't have to calculate SD, but that stuff is pretty "high yield."

3) What materials you used for each section (Kaplan, TPR, Examkrackers, AAMC, TBR, etc)

Content review:

  1. Kaplan complete 7-book series 1st edition. This was written and published in 2014 before the first 2015 MCAT was released.
  2. Khan Academy videos
  3. Several of the free study guides found on the MCAT subreddit (which was VERY helpful)
Practice:
  1. Online AAMC practice materials (the entire online bundle)
  2. Jack Westin (the free daily CARS passages, as well as 2 FL CARS practice exams)
  3. UWorld. May I say that UWorld is probably the MOST HELPFUL 3rd party practice material (wish you could mix/match topics to create pseudo-FL exams.) Because I work full time, I could spare the $200 for 3 month access. The explanations are 100x better than AAMC, while having the same format as the real MCAT. The explanations from UWorld can be anywhere from 3-6 paragraphs long and tell you why every other answer is incorrect. Seriously, it looks like they get their figures and explanations from a text book. You can even make flashcards and track your progress. If you can find an extra $200, buy this if you're serious about getting a great score through practice.
4) Which practice tests did you use? (Optional: include scores)

6 total:
AAMC Sample Test and FL1-3. ALso used NextStep FL1 and 2.

Sample test: 504
AAMC 1: 501
AAMC 2: 507 (gained confidence in myself here)
NS1: 506
NS2: 508
AAMC 3: 509

5) What was your undergraduate major?

Biology with a minor in psychology and biochemistry at a state school.

6) Any other tips you may have for those of us who still have this test lurking over us?

This is for everyone: have confidence in yourself. My friends were studying at the same time as I was and we took FL exams on the same Saturday each week. They were coming out the gate with 513, 515, etc. While I got a 504 on the sample test. Then I got a 501. I wanted to quit because I couldn't stop comparing myself to other people. I already had a 498 (126/121/126/124), which only made my confidence worse. Then, my girlfriend told me "just pretend you're going to score like your friends will. Have confidence in yourself that you can get where you want." My score jumped up 6 points when I only reviewed a few chapters and did (maybe) 40 total practice questions that week. Confidence is KEY. When I truly believed in my abilities, I wasn't getting below a 125 on any subsection.

Also, you cannot look at everything as "high vs. low yield." Pretend everything is fair game. There were at least 3 questions per passage (other than CARS) that had stuff people would consider "low yield." I hated physics and everything about it. But I put myself in the hell hole by forcing myself to study and understand it. I worked my butt off to make sure I could answer SB C/P quality questions.

For my people who have to work PT/FT while studying: you are going to have to work harder and longer than the premeds who studying 8+ hours/day during the summer while volunteering 6-12 hours a week. I hate that I messed up in college with my MCAT, but that was my reality. I went to my wits end to prep for this MCAT. And that's okay. I spent 5.5 months studying, practicing, and crying. It really tested who I was and if I really wanted to go into medicine. If I had to do it again, I'd have taken even longer to study. Realize that people who had nothing to do BUT study could get it done in 2 months. That is not our reality. We will most likely need 4-7 months of dedication to study, practice, and go back to work on weaknesses. But if you have a great support group, hobbies outside of school/medicine/work, and a healthy outlet for the stress, then anything is possible.

7) How long did you study for the MCAT?

5 and a half months while working full time
 
Last edited:
  • Like
Reactions: 1 users
1) Your individual scores and composite score:

Total:
512
C/P: 127
CARS: 128
B/B: 130
P/S: 127

Exam date: 5/18/2019

2) How I studied

So, I work full time at a mentally demanding job doing research. I work anywhere from 40-70 hours/week. I would work out at 4 am during the week and study as soon as I got home, Mon - Thurs. Friday I took off to spend time with my GF (who is also studying the MCAT) to not talk about/look at MCAT stuff. It was a much-needed break from studying. Then, I would spend 7+ hours/day every weekend practicing, studying, and (re)reading chapters/watching KA videos on weaker subjects. I began studying in December off and on; then I got very focused beginning of January.

Chem/Phys: This was my weakest subject by far. When I took a half-length Kaplan diagnostic, I made a 492 composite with a 121 on C/P. I focused on REALLY concentrating on learning the material by reading through Kaplan books. Once I finished those, I would practice every day. I started with AAMC materials then moved to UWorld. I didn't use up all of my QPacks from AAMC but I used up UWorld for everything but Physics. If my day was set for Gen Chem, OChem, etc. I would double my practice and watch all KA videos on the subject if I didn't 100% understand it. My C/P score jumped from a 121 to a 130 on AAMC FL practice exams. A little bummed I couldn't reproduce that come test day; however, I'm happy with my score nonetheless.

CARS: This one was tough for me. I started practicing with Jack Weston free daily passages after trying out Kaplan's method for tackling CARS passages. Every day, I would do 3+ passages, first thing, until I was comfortable with reading to understand CARS. After a while, the *note taking* part of the Kaplan method became second nature to me. I would be able to sum up what each paragraph had talked about in my head, rather than on paper (which saved me time to answer questions.) This was up and down for me because, some days, I'd get 90% correct on AAMC QPack 1 and/or 2. Other days, I would sit at ~55% correct. Eventually, I would complete AAMC CARS QP2 twice and QP1 once. I also trucked through UWorld CARS questions quickly. One thing I also did was read a book for fun each day for 30 minutes to an hour. This got my brain into reading mode before I practiced CARS.

The best advice I was ever given for this section was: you already know how to read. Just pay attention to everything said and 2/3 of the battle is already done (Comprehension and RWT questions.) Now, practice RBT questions and you're golden. This advice carried me to test day. My FL practice scores ranged from 122-129 with no clear trend. So a 128 on the real deal is phenomenal for me. If you're to take away anything from this, it is: PRACTICE THIS EVERY DAY. I didn't read for fun or anything as a kid; also, my ADHD gave me horrible habits - from skimming around when I would get bored to being easily distracted in general. In college, I read only enough to get A's on my papers. I was scoring 1/6 to 2/7 per passage. Hounding on CARS each day while learning why and how I missed each question brought me from 2/7 to 6/7 per passage. A 128 on my exam was 90th %ile. So if I could get that, so can you.

Bio/Biochem:
To be honest, this section was always easy for me. I got a B.S. in Biology with making a 3.7+ GPA taking nothing but bio/chem classes my junior and senior year. On top of that, I did research as a tech for 2.5 years in undergrad and have been at my research job for almost an additional 2. I read scientific journals almost each week and attend plenty of research lectures/talks a few times per month. I feel like my education always trained me for this section. There were definitely some content gaps for me; but for the most part, I didn't study this one too much. My lowest score on FL practice tests was a 127, and my highest was a 131. If I could go back, however, I would definitely study more of lab techniques and more micro (genetics, prokaryotic biology, etc.) That stuff is a big part of the MCAT. I also wish I utilized more of my materials here too. It wasn't because I thought I was "too good" for that. However, I prioritized my weak areas (C/P and CARS) and just left B/B for any time left before bed (unless I had B/B assigned to practice/study that day.)

Psych/Soc: This section was hard for me, despite being 2 classes away from having a major in psychology. What I should have done was went through premed95's deck and memorize the terms. Kaplan was not a big help. It taught it conceptually (which would have been fine if P/S wasn't just a term test.) I would have probably gotten a 130-132 on this section if I just spent the time to read and memorize the terms. What I actually did was read the Kaplan book and watched KA videos over and over. With a ton of practice from AAMC and QWorld. My friend went through premed95's deck and made a 132 on P/S. She memorized the deck, quizzed herself twice on all the terms, and then went into the test. That was it (other than reading through the EK book a little.)

3) What materials you used for each section (Kaplan, TPR, Examkrackers, AAMC, TBR, etc)

Content review:

  1. Kaplan complete 7-book series 1st edition. This was written and published in 2014 before the first 2015 MCAT was released.
  2. Khan Academy videos
  3. Several of the free study guides found on the MCAT subreddit (which was VERY helpful)
Practice:
  1. Online AAMC practice materials (the entire online bundle)
  2. Jack Westin (the free daily CARS passages, as well as 2 FL CARS practice exams)
  3. UWorld. May I say that UWorld is probably the MOST HELPFUL 3rd party practice material (wish you could mix/match topics to create pseudo-FL exams.) Because I work full time, I could spare the $200 for 3 month access. The explanations are 100x better than AAMC, while having the same format as the real MCAT. The explanations from UWorld can be anywhere from 3-6 paragraphs long and tell you why every other answer is incorrect. Seriously, it looks like they get their figures and explanations from a text book. You can even make flashcards and track your progress. If you can find an extra $200, buy this if you're serious about getting a great score through practice.
4) Which practice tests did you use? (Optional: include scores)

6 total:
AAMC Sample Test and FL1-3. ALso used NextStep FL1 and 2.

Sample test: 504
AAMC 1: 501
AAMC 2: 507 (gained confidence in myself here)
NS1: 506
NS2: 508
AAMC 3: 509

5) What was your undergraduate major?

Biology with a minor in psychology and biochemistry at a state school.

6) Any other tips you may have for those of us who still have this test lurking over us?

This is for everyone: have confidence in yourself. My friends were studying at the same time as I was and we took FL exams on the same Saturday each week. They were coming out the gate with 513, 515, etc. While I got a 504 on the sample test. Then I got a 501. I wanted to quit because I couldn't stop comparing myself to other people. I already had a 498 (126/121/126/124), which only made my confidence worse. Then, my girlfriend told me "just pretend you're going to score like your friends will. Have confidence in yourself that you can get where you want." My score jumped up 6 points when I only reviewed a few chapters and did (maybe) 40 total practice questions that week. Confidence is KEY. When I truly believed in my abilities, I wasn't getting below a 125 on any subsection.

Also, you cannot look at everything as "high vs. low yield." Pretend everything is fair game. There were at least 3 questions per passage (other than CARS) that had stuff people would consider "low yield." I hated physics and everything about it. But I put myself in the hell hole by forcing myself to study and understand it. I worked my butt off to make sure I could answer SB C/P quality questions.

For my people who have to work PT/FT while studying: you are going to have to work harder and longer than the premeds who studying 8+ hours/day during the summer while volunteering 6-12 hours a week. I hate that I messed up in college with my MCAT, but that was my reality. I went to my wits end to prep for this MCAT. And that's okay. I spent 6.5 months studying, practicing, and crying. It really tested who I was and if I really wanted to go into medicine. If I had to do it again, I'd have taken even longer to study. Realize that people who had nothing to do BUT study could get it done in 2 months. That is not our reality. We will most likely need 4-7 months of dedication to study, practice, and go back to work on weaknesses. But if you have a great support group, hobbies outside of school/medicine/work, and a healthy outlet for the stress, then anything is possible.

7) How long did you study for the MCAT?

5 and a half months while working full time
Thanks for posting. Very helpful.

What do RBT and RWT mean? You used these terms when discussing CARS strategy.

Also, of the various third party CARS resources you used, which were the closest to AAMCAS CARS?

Appreciate the insights.
 
Thanks for posting. Very helpful.

What do RBT and RWT mean? You used these terms when discussing CARS strategy.

Also, of the various third party CARS resources you used, which were the closest to AAMCAS CARS?

Appreciate the insights.
RWT = Reading Within the Text
RBT = Reading Beyond the Text

Those are two of the three question types you'll see in CARS.

As far as 3rd party,

Jack Westin was hit or miss. I used it as pure practice for reading for comprehension. It doesn't hurt since, if anything, it will just be good practice to read passages. And he has free FL CARS you can take. They are pretty hard and only gives a raw score of % correct.

The best 3rd party would be UWorld since you don't have to use a FL to test your abilities. The only downside is it only has around 250 questions. Pairing it with AAMC and JW, you wont really run out of practice.

Hope this helps!
 
  • Like
Reactions: 1 user
Thanks for posting. Very helpful.

What do RBT and RWT mean? You used these terms when discussing CARS strategy.

Also, of the various third party CARS resources you used, which were the closest to AAMCAS CARS?

Appreciate the insights.
On thing I'd like to add to UWorld:

Their passages were mostly easy to read and comprehend. That doesn't always translate to the true MCAT, where I easily had 2-4 passages that were dense and difficult to follow. That is the only downside.

HOWEVER, the questions they ask are pretty hard. I would finish a passage and think "oh man, that was pretty easy to follow. Bring it on." Then, I'd get wrecked by the questions because it really teases out how much you understood what the author was trying to say. It really makes you think like how the AAMC wants you to think as a 130-132 (in CARS) test taker.
 
  • Like
Reactions: 1 user
Hey everyone, saw this thread and wanted to contribute in hopes that someone else might find this helpful!

1) Your individual scores and composite score:
5/24/19 - 516 (131/125/130/130)

2) The study method used for each section
Overall, for all content sections, I had a specific process I figured out halfway through my time studying that worked well for me. Given more time, I think I would have been able to score a bit higher using this method, at least for me. The process was as follows: First I suspended the whole ortho deck at the start of studying for a section. Then, I would read an individual Kaplan book chapter, and then unsuspend the topics that I'd just covered in the Kaplan book from the ortho deck and do my Anki for the day. The NEXT day, Id do the associated UWorld with those sections, then repeat the process. Over time, you'll be adding more and more ortho and compounding onto that deck as you unsuspend more topics. I mostly used this for B/B and P/S, using the ortho deck for the Anki part of this process for the B/B and ortho/cubene for the P/S.

C/P - For C/P I did all of UWorld by section within a topic, not random, so that I could see where my weaknesses were. Id then go into my books to find and clarify certain answers using UWorld along hte way. I did section bank very early, with 1.5 months left, then did it again a week before the exam.

CARS - I cant help you here lol. Didn't study for it.

B/B - So initially I had problems getting questions right on UWorld, so I resolved to go through the entire Kaplan bio book taking notes in question format. I would then go down these lists of questions to make sure I knew. My percentages went from 50-60 to around 85-90 from this method on UWorld. Again, I redid section bank a week before the exam. I never made my own Anki cards, I used ortho528s deck in the aforementioned strategy.

P/S - I think this is pretty well known, but I watched the KA vids at 2x speed while reading the 300 page doc and then did the Anki associated with that section. UWorld next day. Section bank was also done twice at the same time as those previous sections.

3) What materials you used for each section(Kaplan, TPR, ExamKrackers, AAMC, TBR, etc)
Kaplan, AAMC, UWorld, NS for all.

4) Which practice tests did you use? (Optional: include scores)
NSDiag - 498 (125/123/123/127) - Taken while running an experiment in the lab, untimed.
NS1 - 507 (128/125/127/127)
NS2 - 511 (128/126/130/127)
NS3 - 510 (127/126/128/129)
NS4 - 511 (129/125/129/128)

AAMC FL1 - 519 (131/127/131/130)
AAMC FL2 - 520 (132/128/130/130)
AAMC FL3 - 521 (131/128/131/131)
Sample - Taken as a question bank the day before the exam, got around 93-100% on the content sections.

5) What was your undergraduate major?
Biomedical Engineering.

6) Any other tips you may have for those of us who still have this test lurking over us?
Study CARS early, no matter the NS score. I fell into the trap of "It'll work itself out, its just NS and its garbage CARS" but it clearly was quite indicative. Even in FL1-3 I thought I was a bit shaky and got somewhat lucky with 128s but I didn't study for it at all just because I thought in the 3 week timeframe I had it was low yield.

7) How long did you study for the MCAT?
3.5 months while being a full time student and working in the lab.
 
  • Like
Reactions: 2 users
Two words: Youtube videos and Uworld.

1) 514 (128, 125 (!), 129, 132) 91 percentile. First time taking it.

FL 1: 512
FL 2: 515
FL 3: 511

Background: 3.4 GPA, 3.27 science. I went to a public research university (not a top 20). Lots of B's in science classes (and C in ochem+biochem). I felt like I understood the science content but for some reason wouldn't test well. If you struggled with science classes do not worry, if I did it you will be able to :)

Classes that (I think) helped me for the test: physiology (even tho I had to relearn a lot of it), biochem, cell biology.

2)
C/P: Oh boy this is my worst section. I read all of the EK books for this and it was very helpful, however, the material FINALLY clicked when I started watching Pre-Med HQ videos on youtube. His videos were straight to the point and so concise that it finally made sense. This guy needs more viewers and is so underrated. I tried to watch AK and KA but the videos were so long and painfully detailed that I would get lost. After content I practiced LIKE CRAZY using all the AAMC q packs and section banks. After I finished those I worked on the uworld passages and finished all of physics and ochem questions. Practice is key here and doing scientific notations for the calculations.

B/B: This section is usually my strength and I ranged from 130-131 on the AAMC tests. I read the EK books and did a lot of practice problems. I did the section bank 3 times and that really helped me on test day (I def think you should redo these at least 3x for each section and try to not use prior knowledge when answering it). I started to see patterns and on test day I could anticipate what questions they were going to ask from the passage. Also EK didn't cover metabolism in detail so I would recommend watching YT videos on glycolysis, Krebs, ETC and to less detail gluconeogenesis, glycogensis, etc (have a good idea but don't kill yourself with small details, just know the use and the important enzyme for each).

CARS: I suck at this because english is not my first language. CARS I practiced reading slowly and then answering questions (but obviously it didn't help on test day). I didnt really see improvements while I studied so I started to emphasize this section less. Test day was the lowest I scored (I usually got around 127-128 on practice AAMC). The weird thing is that on test day the passages were the easiest I had read. Oh well!

P/S: This was the lowest scored section while I was practicing. 5 days before my test I scored 127 and a week before 126! The last 5 days I reread the 86 page document and did ALL the uworld P/S questions over the last 3 weeks which SAVED my ass test day. If I didn't do uworld there's no way I would have gotten a 132.


4) I used NS 1-4 and ranged from 507-509 at the end of my studying (I took them a few months prior but didnt look at solution, I was scoring 500-503 then). I would use them again even though they were tricky. They made AAMC feel like a breeze once I switched over to the AAMC practice content.

5) Biology. I was a Biology teacher too which helped with my foundation (mitosis/meiosis, evolution, organelles, punnett squares, etc)

6) Practice for at least 6-8 weeks, you'll be reviewing content anways as you practice. I feel that content is 40% and is 60% practice. So content is pretty important but not if you can't apply it to the test. Through practice you get comfortable seeing which questions will be asked and some of the things you need to learn aren't explicitly taught in the books. Use reddit when reviewing your tests (the solutions in AAMC suck). Don't take the test until you're ready. I rescheduled my test 3x until I felt ready.

Also, test in testing conditions. I would get anxious and nervous during my practice tests so I practiced breathing and meditation before the tests. I'm not a yogi or anything but it helped on test day because I was super calm and focused. Also go to the testing center the day before to calm your nerves and get familiar with the site. And for the love of god SLEEP! I got a good 7 hours of sleep the night before. I can't imagine not sleeping like some of these other testers. Also during the test if you're spending too long on the questions flag it and move ON! You will have extra time at the end and you can go back and look with a fresh pair of eyes. I did this on test day and I was able to figure out the problems quickly once I took a little break.

7) Started in January so 5 months part time, few hours M-F, 5-6 hours on weekends (I was working 40-50 hours a week). I took a break in March for a week to travel. From May to end of June I studied 1 month full time.


Don't stress, take your time, YOU CAN DO IT.

I know this was a year ago but I am curious, did you watch all of premed HQ videos or just his MCAT ones? Chem is easily my weakest section and I want to bring it up greatly.
 
Top